SlideShare a Scribd company logo
1 of 30
Download to read offline
CÁC PHƯƠNG PHÁP GI I PHƯƠNG TRÌNH HÀM THƯ NG DÙNG
Phương pháp 1: H s b t ñ nh.
Nguyên t c chung:
+) D a vào ñi u ki n bài toán, xác ñ nh ñư c d ng c a f(x), thư ng là f(x) = ax + b ho c
f(x) = ax2+ bx + c.
+) ð ng nh t h s ñ tìm f(x).
+) Ch ng minh r ng m i h s khác c a f(x) ñ u không th a mãn ñi u ki n bài toán.
Ví d 1: Tìm f : R → R th a mãn: f ( x f ( y ) + x ) = xy + f ( x ) ∀x, y ∈ R (1) .
L i gi i:
     x = 1
Thay       vào (18) ta ñư c: f ( f ( y ) + 1) = y + f (1) ( a ) .
     y∈R

                                                    (               )
Thay y = − f (1) − 1 vào (a) suy ra: f f ( − f (1) − 1) + 1 = −1 . ð t a = f ( − f (1) − 1) + 1 ta

ñư c: f ( a ) = −1 .

     y = a
Ch n       ta ñư c: f ( x f ( a ) + x ) = xa + f ( x ) ⇒ xa + f ( x ) = f ( 0 ) .
     x ∈ R
ð t f ( 0 ) = b ⇒ f ( x ) = −a x + b . Th vào (1) và ñ ng nh t h s ta ñư c:

                  a = 1
 a 2 = 1                      f ( x) = x
               ⇒   a = −1 ⇒ 
                                            .
 − a b − a = −a               f ( x) = −x
                               
                  b = 0
V y có hai hàm s c n tìm là f ( x ) = x và f ( x ) = − x .

Ví d 2: Tìm f : R → R th a mãn: f ( f ( x ) + y ) = y f ( x − f ( y ) ) ∀x, y ∈ R ( 2 ) .
L i gi i:
Cho y = 0; x ∈ R : (2) ⇒ f ( f ( x ) ) = 0 ∀x ∈ R ( a ) .

                              (                 )
Cho x = f ( y ) : (2) ⇒ f f ( f ( y ) ) + y = y f ( 0 ) ( a ' ) .

( a ) + ( a' ) ⇒ f ( y ) = y f ( 0) . ð   t f ( 0 ) = a ⇒ f ( y ) = ay ∀y ∈ R . Th l i (2) ta ñư c:

 a 2 ( x 2 + y 2 ) + a ( y − x y ) = 0 ∀x, y ∈ R ⇔ a = 0 ⇒ f ( x ) = 0 ∀x ∈ R . V y có duy nh t hàm s

 f ( x ) = 0 th a mãn bài toán.
Ví d 3: Tìm f , g : R → R th a mãn:

              2 f ( x ) − g ( x ) = f ( y ) − y ∀x, y ∈ R
                                                              (a)
                                                                   .
               f ( x) g ( x) ≥ x +1
                                                ∀x ∈ R        (b )
L i gi i:
Cho x = y ∈ R khi ñó ( a ) ⇒ f ( x ) = g ( x ) − x .Thay l i (a) ta ñư c:

                                                        1
g ( x ) = 2 x − 2 y + g ( y ) ∀x, y ∈ R (c).

Cho y = 0; x ∈ R : t (c) ta ñư c: g ( x ) = 2 x + g ( 0 ) . ð t g ( 0 ) = a ta ñư c:

 g ( x ) = 2 x + a , f ( x ) = x + a . Th vào (a), (b) ta ñư c:

               2 x + a = 2 x + a
               
    (a), (b) ⇔                              ( ∀x ∈ R ) ⇔ 2 x 2 + ( 3a − 1) x + a 2 − 1 ≥ 0 ∀x ∈ R
               (
                 x + a )( 2 x + a ) ≥ x + 1
                  2
    ⇔ ( a − 3) ≤ 0 ⇔ a = 3 . V y f ( x ) = x + 3 ; g ( x ) = 2 x + 3 .

Ví d 4: ða th c f(x) xác ñ nh v i ∀x ∈ ℝ và th a mãn ñi u ki n:
            2 f ( x ) + f (1 − x ) = x 2 , ∀x ∈ ℝ (1). Tìm f(x).
L i gi i:
Ta nh n th y v trái c a bi u th c dư i d u f là b c nh t: x, 1 – x v ph i là b c hai x2.
V y f(x) ph i có d ng: f(x) = ax2 + bx + c.
Khi ñó (1) tr thành: 2(ax2 + bx + c) + a(1 – x)2 + b(1 – x) + c = x2 ∀x ∈ ℝ do ñó:
3ax2 + (b – 2a)x + a + b + 3c = x2, ∀x ∈ ℝ
                                                       1
                                                   a = 3
                                 3a = 1           
                                                      2
ð ng nh t các h s , ta thu ñư c: b − 2a = 0     ⇔ b =
                                 a + b + 3c = 0       3
                                                        1
                                                   c = − 3
                                                   
             1
V y: f ( x) = ( x 2 + 2 x − 1)
             3
Th l i ta th y hi n nhiên f(x) th a mãn ñi u ki n bài toán.
Ta ph i ch ng minh m i hàm s khác f(x) s không th a mãn ñi u ki n bài toán:
Th t v y gi s còn hàm s g(x) khác f(x) th a mãn ñi u ki n bài toán.
Do f(x) không trùng v i g(x) nên ∃x0 ∈ ℝ : g ( x0 ) ≠ f ( x0 ) .

Do g(x) th a mãn ñi u ki n bài toán nên: 2 g ( x) + g (1 − x) = x 2 , ∀x ∈ ℝ

Thay x b i x0 ta ñư c: 2 g ( x0 ) + g (1 − x0 ) = x0 2

Thay x b i 1 –x0 ta ñư c: 2 g (1 − x0 ) + g ( x0 ) = (1 − x0 ) 2
                                    1
T hai h th c này ta ñư c: g ( x0 ) = ( x0 2 + 2 x0 − 1) = f ( x0 )
                                    3
ði u này mâu thu n v i g ( x0 ) ≠ f ( x0 )
                                               1
V y phương trình có nghi m duy nh t là f ( x) = ( x 2 + 2 x − 1)
                                               3



                                                        2
Nh n xét: N u ta ch d ñoán f(x) có d ng nào ñó thì ph i ch ng minh s duy nh t c a các
hàm s tìm ñư c.
Ví d 5: Hàm s y = f(x) xác ñ nh, liên t c v i ∀x ∈ ℝ và th a mãn ñi u ki n:
                        f(f(x)) = f(x) + x, ∀x ∈ ℝ
Hãy tìm hai hàm s như th .
L i gi i:
Ta vi t phương trình ñã cho dư i d ng f(f(x)) – f(x) = x (1).
V ph i c a phương trình là m t hàm s tuy n tính vì v y ta nên gi s r ng hàm s c n tìm
có d ng: f(x) = ax + b.
Khi ñó (1) tr thành: a( ax + b) + b – (ax + b) = x , ∀x ∈ ℝ hay (a2 –a )x + ab = x, ∀x ∈ ℝ
                                              
                       a 2 − a = 1 a = 1 + 5 a = 1 − 5           1± 5
ñ ng nh t h s ta ñư c:            ⇔       2 ∨       2 ⇒ f ( x) =      x.
                       ab = 0      b = 0     b = 0                 2
                                              
Hi n nhiên hai hàm s trên th a mãn ñi u ki n bài toán (vi c ch ng minh s duy nh t dành
cho ngư i ñ c).
Ví d 6: Hàm s        f : ℤ → ℤ th a mãn ñ ng th i các ñi u ki n sau:
      a ) f ( f ( n)) = n, ∀n ∈ ℤ         (1)
      b) f ( f (n + 2) + 2) = n, ∀n ∈ ℤ    (2)
      c) f (0) = 1                         (3)
Tìm giá tr f(1995), f(-2007).
L i gi i:
Cũng nh n xét và lý lu n như các ví d trư c, ta ñưa ñ n f(n) ph i có d ng: f(n) = an +b.
Khi ñó ñi u ki n (1) tr thành: a 2 n + ab + b = n, ∀n ∈ ℤ

                             a 2 = 1     a = 1 a = −1
ð ng nh t các h s , ta ñư c:            ⇔     ∨
                             ab + b = 0  b = 0 b = 0
    a = 1
V i       ta ñư c f(n) = n. Trư ng h p này lo i vì không th a mãn (2).
    b = 0
    a = −1
V i        ta ñư c f(n) = -n + b. T ñi u ki n (3) cho n = 0 ta ñư c b = 1.
    b = 0
V y f(n) = -n + 1.
Hi n nhiên hàm s này th a mãn ñi u ki n bài toán.
Ta ph i ch ng minh f(n) = -n +1 là hàm duy nh t th a mãn ñi u ki n bài toán:
Th t v y gi s t n t i hàm g(n) khác f(n) cũng th a mãn ñi u ki n bài toán.
T (3) suy ra f(0) = g(0) = 1, f(1) = g(1) = 0.
S d ng ñi u ki n (1) và (2) ta nh n ñư c: g(g(n)) = g(g(n+2)+2) ∀n ∈ℤ .


                                                 3
do ñó g(g(g(n))) = g(g(g(n+2)+2)) ∀n ∈ℤ Hay g(n) = g(n+2)+2 ∀n ∈ℤ .

Gi s n0 là s t nhiên bé nh t làm cho f (n0 ) ≠ g (n0 )
Do f(n) cũng th a mãn (4) nên ta có:
g (n0 − 2) = g (n0 ) + 2 = f (n0 ) + 2 = f (n0 − 2)
⇔ g (n0 − 2) = f (n0 − 2)
Mâu thu n v i ñi u ki n n0 là s t nhiên bé nh t th a mãn (5).
V y f(n) = g(n), ∀n ∈ ℕ
Ch ng minh tương t ta cũng ñư c f(n) = g(n) v i m i n nguyên âm.
V y f(n) = 1 – n là nghi m duy nh t.
T ñó tính ñư c f(1995), f(-2007).
                                                     BÀI T P
Bài 1: Tìm t t c các hàm s            f : ℝ → ℝ th a mãn ñi u ki n:

f ( x + y ) + f ( x − y ) − 2 f ( x) f (1 + y ) = 2 xy (3 y − x 2 ), ∀x, y ∈ ℝ .

ðáp s : f(x) = x3.
Bài 2: Hàm s       f : ℕ → ℕ th a mãn ñi u ki n f(f(n)) + f(n) = 2n + 3, ∀n ∈ ℕ. Tìm f(2005).
ðáp s : 2006.
Bài 3: Tìm t t c các hàm f : ℕ → ℕ sao cho: f ( f (n)) + ( f (n))2 = n 2 + 3n + 3, ∀n ∈ ℕ.
ðáp s : f(n) = n + 1.
                                       x −1        1− x     8             2      
Bài 4: Tìm các hàm f : ℝ → ℝ n u: 3 f         −5f        =     , ∀x ∉ 0, − ,1, 2 
                                       3x + 2      x − 2  x −1            3      
                    28 x + 4
ðáp s : f ( x) =
                      5x
Bài 5: Tìm t t c các ña th c P(x) ∈ ℝ [ x] sao cho: P(x + y) = P(x) + P(y) + 3xy(x + y),
∀x , y ∈ ℝ
ðáp s : P(x) = x3 + cx.
Phương pháp 2: phương pháp th .
2.1. Th      n t o PTH m i:
                                      2x +1 
Ví d 1: Tìm f: R{2} → R th a mãn: f         = x + 2 x ∀x ≠ 1 (1) .
                                                  2

                                       x −1 

                   2x +1 
L i gi i: ð t t =         ⇒ MGT t = R  {2} (t p xác ñ nh c a f). Ta ñư c:
                   x −1      x ≠1


     t +1                       3t 2 − 3
x=        th vào (1): f (t ) =          2
                                          ∀t ≠ 2 . Th l i th y ñúng.
     t−2                       (t − 2)


                                                           4
3x 2 − 3
V y hàm s c n tìm có d ng f ( x) =                      2
                                                            .
                                             ( x − 2)
Nh n xét:
+ Khi ñ t t, c n ki m tra gi thi t MGT t ⊃ D . V i gi thi t ñó m i ñ m b o tính ch t: “Khi
                                             x∈Dx

t ch y kh p các giá tr c a t thì x = t cũng ch y kh p t p xác ñ nh c a f”.
                                                               3x 2 − 3
                                                                         2            ( x ≠ 2)
+ Trong ví d 1, n u f: R → R thì có vô s hàm f d ng: f ( x) =  ( x − 2 )                         (v i a∈R
                                                              
                                                              a                       ( x = 2)
tùy ý).
Ví d 2: Tìm hàm f : ( −∞; −1] ∪ ( 0;1] → R th a mãn:

          f ( x − x 2 − 1) = x + x 2 − 1 ∀ x ≥ 1 ( 2 ) .

                                                  x − t ≥ 0
                                                  
L i gi i: ð t t = x − x 2 − 1 ⇔ x 2 − 1 = x − t ⇔  2               2
                                                  x −1 = ( x − t )
                                                  
                           x ≥ t
 x ≥ t                                                t2 +1      t ≤ −1
⇔ 2        2           2
                          ⇔      2
                                t +1  . H có nghi m x ⇔       ≥t ⇔ 
  x − 1 = x − 2 xt + t    x =                           2t       0 < t ≤ 1
                                  2t
⇒ t ∈ ( −∞; −1] ∪ ( 0;1] . V y MGT t = D = ( −∞; −1] ∪ ( 0;1] .
                                     x ≥1


                                             1           1
V i t = x − x 2 − 1 thì x + x 2 − 1 =          ⇒ f (t ) = th a mãn (2).
                                             t           t
               1
V y f ( x) =     là hàm s c n tìm.
               x
                   2                    3x − 1  x + 1
Ví d 3: Tìm f : R  ;3 → R th a mãn: f         =      ∀x ≠ 1, x ≠ −2 ( 3) .
                   3                    x + 2  x −1
                      3x − 1               2         2t + 1                              t+4
L i gi i: ð t t =            ⇒ MGT t = R   ;3 ⇒ x =        th vào (4) ta ñư c: f (t ) =
                      x+2      ( x ≠2)
                                 x ≠1
                                           3         3−t                                 3t − 2

                                                             x+4
th a mãn (3). V y hàm s c n tìm là: f ( x) =                       .
                                                            3x − 2
Ví d 4: Tìm f :       ( 0; + ∞ ) → ( 0; + ∞ )   th a mãn:

          x f ( x f ( y )) = f ( f ( y )) ∀x, y ∈ ( 0; + ∞ ) (4) .

L i gi i:
Cho y = 1, x ∈ ( 0; + ∞ ) ta ñư c: x f ( x f (1)) = f ( f (1)) .

              1                                                              1
Cho x =           ta ñư c: f ( f (1) = 1⇒ x f ( x f (1)) = 1 ⇒ f ( x f (1)) = . ð t:
            f (1)                                                            x

                                                                5
f (1)           a
t = x. f (1) ⇒ f (t ) =         ⇒ f (t ) = (v i a = f (1) ). Vì f (1) ∈ ( 0; + ∞ ) ⇒ MGT t = ( 0; + ∞ ) .
                            t             t                                          x∈( 0; +∞ )


               a                                                          a
V y f ( x) =     . Th l i th y ñúng ( a > 0 ) . Hàm s c n tìm là: f ( x) = v i ( a > 0 ) .
               x                                                          x
Ví d 5: Tìm hàm f: ( 0; + ∞ ) → ( 0; + ∞ ) th a mãn:

                 1                       3                 3
          f (1) = ; f ( xy ) = f ( x). f   + f ( y ). f      ∀x, y ∈ ( 0; + ∞ ) ( 5 ) .
                 2                        y                x
L i gi i:
                                             1
Cho x = 1; y = 3 ta ñư c: f ( 3) =             .
                                             2
                                               3
Cho x = 1; y ∈ ( 0; + ∞ ) ta ñư c: f ( y ) = f   . Th l i (5) ta ñư c:
                                                y
                                                                          3
 f ( xy ) = 2 f ( x) f ( y ) ∀x, y ∈ ( 0; + ∞ ) (5') . Thay y b i           ta ñư c:
                                                                          x
                                         2
                       3   1              2
 f ( 3) = 2 f ( x )) f   ⇒   = ( f ( x ) ) . Th l i th y ñúng.
                       x   2
                                         1
V y hàm s c n tìm là: f ( x ) =            ∀x > 0 .
                                         2
Ví d 6: Tìm hàm f: R → R th a mãn:

         ( x − y ) f ( x + y ) − ( x + y ) f ( x − y ) = 4 xy ( x 2 + y 2 )       ∀x, y ∈ R   ( 6) .
L i gi i: Ta có:
( 6) ⇔ ( x − y ) f ( x + y ) − ( x + y ) f ( x − y ) =
                                                       1                        2 1                          2
 = ( x + y ) − ( x − y )  + ( x + y ) + ( x − y )   ( x + y ) + ( x − y )  −  ( x + y ) − ( x − y )  
                                                   4                        4                        
                                                                                                              
    u = x − y                                 1
ð t 
    v = x + y
                                                                            2
                                                                              (
               ta ñư c: v f ( u ) − u f ( v ) = ( u + v )( u − v ) ( u + v ) − ( u − v )
                                               4
                                                                                         2
                                                                                                       )
⇒ v f ( u ) − u f ( v ) = u 3v − v 3u ⇔ v ( f ( u ) − u 3 ) = u ( f ( v ) − v3 )

+ V i uv ≠ 0 ta có:
 f ( u ) − u 3 f ( v ) − v3               f (u ) − u3
              =             ∀u , v ∈ R* ⇒             = a ⇒ f ( u ) = au + u 3 ∀u ≠ 0 .
      u             v                         u
+ V i u = 0; v ≠ 0 suy ra: f ( u ) − u 3 = 0 ⇔ f ( u ) = u 3 ⇒ f ( 0 ) = 0 .

Hàm f ( u ) = au + u 3 th a mãn f ( 0 ) = 0 . V y f ( u ) = au + u 3 ∀u ∈ R

Hàm s c n tìm là: f ( x ) = ax + x3 ( a ∈ R ) . Th l i th y ñúng.

2.2. Th      n t o ra h PTH m i:

                                                            6
Ví d 1: Tìm hàm f: R → R th a mãn: f ( x ) + x f ( − x ) = x + 1 ∀x ∈ R (1) .

L i gi i:
ð t t = − x ta ñư c: f ( −t ) − t f ( t ) = −t + 1 ∀t ∈ R (1) . Ta có h :

 f ( x) + x f (−x) = x +1

                                  ⇒ f ( x ) = 1 . Th l i hàm s c n tìm là: f ( x ) = 1 .
− x f ( x ) + f ( − x ) = − x + 1

                                                             x −1 
Ví d 2: Tìm hàm s f : R  { 0,1 } → R Th a mãn: f ( x ) + f        = 1 + x ∀x ∈ R
                                                                                    *
                                                                                                    ( 2) .
                                                             x 
                        x −1
L i gi i: ð t x1 =           , ( 2 ) ⇔ f ( x ) + f ( x1 ) = 1 + x .
                          x
            x1 − 1     1
ð t x2 =           =      , ( 2 ) ⇔ f ( x1 ) + f ( x2 ) = 1 + x1 .
              x1     x −1
           x2 − 1
ð t x3 =          = x, ( 2 ) ⇔ f ( x2 ) + f ( x ) = 1 + x2 .
             x2

         f ( x1 ) + f ( x ) = 1 + x
                                                  1 + x − x1 + x2 1  1  1 
Ta có h  f ( x2 ) + f ( x1 ) = 1 + x1 ⇒ f ( x ) =                = x+ +      . Th l i th y
                                                         2        2  x 1− x 
         f ( x ) + f ( x2 ) = 1 + x2
                                          1    1  1 
ñúng. V y hàm s c n tìm có d ng: f ( x ) =  x + +     .
                                          2    x 1− x 

                                                                                x −1 
Ví d 3: Tìm hàm s            f : R  { − 1;0;1 } → R th a mãn: x f ( x ) + 2 f        = 1 ∀x ≠ −1 ( 3) .
                                                                                x +1 
L i gi i:
            x −1
ð t x1 =         , ( 3) ⇒ x f ( x ) + 2 f ( x1 ) = 1 .
            x +1
            x1 − 1    1
ð t x2 =           = − , ( 3) ⇒ x 1 f ( x1 ) + 2 f ( x2 ) = 1 .
            x1 + 1    x
            x2 − 1 x + 1
ð t x3 =          =      , ( 3) ⇒ x2 f ( x2 ) + 2 f ( x3 ) = 1 .
            x2 + 1 x − 1
            x3 − 1
ð t x4 =           = x , ( 3) ⇒ x3 f ( x3 ) + 2 f ( x ) = 1 .
            x3 + 1

         x f ( x ) + 2 f ( x1 ) = 1
        
         x1 f ( x1 ) + 2 f ( x2 ) = 1            4 x2 − x + 1
Ta có h                               ⇒ f ( x) =              . Th l i th y ñúng.
         x2 f ( x2 ) + 2 f ( x3 ) = 1            5 x ( x − 1)
        x f x + 2 f x = 1
         3 ( 3)             ( )


                                                           7
4 x2 − x + 1
V y hàm s c n tìm là: f ( x ) =                   .
                                     5 x ( x − 1)

                                                  BÀI T P
                                      1
1) Tìm f : R  { 1 } → R th a mãn: f  1 +  = x 2 + 1 ∀x ∈ R .
                                      x

                a                   b − ax   x2         a
2) Tìm f : R   −  → R th a mãn: f         = 4    ∀x ≠ − (a, b là h ng s cho
                b                   bx + a  x + 1       b
trư c và ab ≠ 0 ).
3) Tìm f : R → R th a mãn: f ( 2002 x − f ( 0 ) ) = 2002 x 2               ∀x ∈ R .

                                                      1  1 
4) Tìm f : R  { 0 } → R th a mãn: f ( x ) +            f        = 1 ∀x ∈ R  { 0;1} .
                                                      2x  1 − x 

                                                     1− x 
5) Tìm f : R  { ± 1; 0} → R th a mãn: ( f ( x ) ) f        = 64 x ∀x ∈ R  {−1} .
                                                      1+ x 
               2                                           2x                  2
6) Tìm f : R    → R th a mãn: 2 f ( x ) +               f         = 996 x ∀x ≠ .
               3                                           3x − 2              3

                                        x −3                  x+3
7) Tìm f : R  { ± 1 } → R th a mãn: f       +              f       = x ∀x ≠ ±1 .
                                        x +1                  1− x 
8) Tìm f : R → R th a mãn: 2 f ( x ) + f (1 − x ) = x 2             ∀x ∈ R .

                                       1
9) Tìm f : R → R th a mãn: f ( x ) + f   = x 2008               ∀x ∈ R* .
                                       x
                 1                            x −1            1
10) Tìm f : R  ±  → R th a mãn: f ( x ) + f          = x ∀x ≠ .
                 3                            1 − 3x          3

                                         a2 
11) Tìm f : R → R th a mãn: f ( x ) + f      = x ∀x ≠ a                       ( a > 0) .
                                        a−x
                                        f            ( 2 x + 1) + 2 g ( 2 x + 1) = 2 x
                                        
12) Tìm f , g : R  { 1 } → R th a mãn:               x        x                     ∀x ≠ 1 .
                                        f                  + g      =x
                                                      x −1     x −1 

Phương pháp 3: Phương pháp chuy n qua gi i h n.
                                                             2 x  3x
Ví d 1: Tìm hàm s f : R → R liên t c, th a mãn: f ( x ) + f   =      ∀x ∈ R (1) .
                                                             3  5
L i gi i:
            2x                             3
ð t x1 =       ; (1) ⇒ f ( x ) + f ( x1 ) = x .
            3                              5
            2 x1                              3
ð t x2 =         ; (1) ⇒ f ( x1 ) + f ( x2 ) = x1 .
             3                                5

                                                       8
2 xn                                           3
ð t xn +1 =          , n ∈ N * ; (1) ⇒ f ( xn ) + f ( xn +1 ) = xn .
                 3                                             5
                              3
         f ( x ) + f ( x1 ) = 5 x                  (1)
        
         f (x )+ f (x ) = 3 x
                                                   ( 2)
               1          2        1
Ta có h                        5
        ……
        
        f x + f x                3
           ( n ) ( n+1 ) = xn                       ( n + 1)
                                 5
Nhân dòng phương trình th (i) v i (-1)i+1 r i c ng l i ta ñư c:

                                      3  2  2           2 
                                                   2           n
                   n+2
 f ( x ) + ( −1)         f ( xn +1 ) = x 1 − +   − ⋯ +  −   ( *) .
                                      5  3  3
                                                          3  
                                                           ( f l.tôc )
Xét lim ( −1) f ( xn +1 )  = lim  f ( xn +1 )  =
              n+ 2
                                                                       f ( lim xn +1 ) = f ( 0 ) .
                          
                                                      n+ 2
M t khác (1) suy ra f(0) = 0 nên lim ( −1)                   f ( xn +1 ) = 0 .

                                                             3   1     9x
L y gi i h n hai v c a (*) ta ñư c: f ( x ) =                  x     =    . Th l i th y ñúng.
                                                             5 1 + 2 25
                                                                   3
                                           9x
V y hàm s c n tìm là: f ( x ) =               .
                                           25
Ví d 2: Tìm hàm s f liên t c t i xo= 0 th a mãn:
            f : R → R và 2 f ( 2 x ) = f ( x ) + x ∀x ∈ R                      ( 2) .
L i gi i:
                                   t t
ð t t = 2 x ta ñư c: 2 f ( t ) = f   + ∀t ∈ R ( 2' ) .
                                   2 2
                 1             *
         tn +1 = 2 tn , ∀n ∈ N
         
Xét dãy:                         . Thay dãy {tn} vào (2’) ta ñư c:
         t = 1 t
         1 2
         

           1            1
 f ( t ) = 2 f ( t1 ) + 4 t         (1)

 f (t ) = 1 f (t ) + 1 t
 1                                  ( 2 ) . Th
            2
                    2
                          4
                             1
                                                  (n) vào ( n − 1) → ( n − 2 ) → ⋯ ta ñư c:
⋯⋯

f t           1             1
 ( n −1 ) 2 ( n ) 4 n −1            (n)
            = f t + t

            1              1                  1                    1
 f (t ) =
            2 n
                f ( tn ) + n +1 f ( tn −1 ) + n f ( tn − 2 ) + ⋯ + 2 t
                          2                  2                    2
                                                                                 (* ) .
                                                                                    '




                                                                  9
n
          1                               1               1 1         1 
Thay tn =   t vào (*’) ta ñư c: f ( t ) = n f ( tn ) + t  2 + 4 + ⋯ + 2 n 
          2                              2               2 2         2 
                                                                                 (* ) .
                                                                                     "




                                  1                                                        t
Vì f liên t c t i xo = 0 nên lim  n f ( tn )  = 0 . L y gi i h n 2 v (*”) suy ra: f ( t ) = . Th
                                 2                                                         3
l i th y ñúng.
Nh n xét:
        +) N u dãy {xn} tu n hoàn thì ta gi i theo phương pháp th r i quy v h pt hàm.
        +) N u dãy {xn} không tu n hoàn nhưng f liên t c t i xo = 0 và {xn} → 0 thì s d ng
gi i h n như VD1.
        + N u {xn} không tu n hoàn, không có gi i h n thì ph i ñ i bi n ñ có dãy {tn} có
gi i h n 0 và làm như ví d 1.
                                                BÀI T P
1) Tìm f : R → R th a mãn:
        a) f liên t c t i xo = 0,
        b) n f ( nx ) = f ( x ) + nx ∀n ∈ N , n ≥ 2; ∀x ∈ R .

                                                               x  10
2) Tìm f : R → R liên t c t i xo = 0, th a mãn: f ( 3 x ) + f   = x .
                                                              3 3
3) Tìm f : R → R liên t c t i xo = 0, th a mãn:

         m f ( mx ) − n f ( nx ) = ( m + n ) x ∀m, n ∈ N * , m ≠ n , ∀x ∈ R .

Phương pháp 4: Phương pháp xét giá tr .
+) ðây là phương pháp cơ s c a m i phương pháp khác.
+) Khi v n d ng phương pháp c n chú ý s d ng k t qu v a có ñư c.
                                ( a ) f ( x ) ≥ 0 ∀x ∈ R
                                
Ví d 1: Tìm f : R → R th a mãn:                                                 .
                                ( b ) f ( x + y ) ≥ f ( x ) + f ( y ) ∀x, y ∈ R
                                
L i gi i:

    x = 0         f ( 0) ≥ 0
                  
Cho       suy ra                    ⇒ f (0) = 0 .
    y = 0         f ( 0) ≥ 2 f ( 0)
                  

               f ( 0) ≥ f ( x ) + f ( − x )  f ( x) + f ( − x ) ≤ 0
                                            
Cho y = − x ⇒                              ⇒
               f ( x ) ≥ 0, f ( − x ) ≥ 0
                                             f ( x ) ≥ 0, f ( − x ) ≥ 0
                                             
⇒ f ( x ) = f ( − x ) = 0 ∀x ∈ R . V y f ( x ) = 0 . Th l i th y ñúng.

Ví d 2: Tìm f : R → R th a mãn:
         1           1                              1
           f ( xy ) + f ( yz ) − f ( x ) f ( yz ) ≥   ∀x, y, z ∈ R    ( 2) .
         2           2                              4
L i gi i:
                                                    10
2
                                                   2    1            1                1
Cho x = z , y = 1 ta ñư c: f ( x ) − ( f ( x ) )       ≥ ⇔  f ( x ) −  ≤ 0 ⇔ f ( x ) = . Th l i th y
                                                        4            2                2
ñúng.
Ví d 3: Tìm f : R → R th a mãn: f ( x ) = Max { xy − f ( y ) } ∀x ∈ R                          ( 3) .
                                                         y∈R


L i gi i: ( 3) ⇒ f ( x ) ≥ xy − f ( y ) ∀x, y ∈ R .

                                    t2
Cho x = y = t ∈ R ⇒ f ( t ) =          ∀t ∈ R   (a) .
                                    2
T (a) suy ra:
                       y2   x2 1        2  x2                                                           x2
 xy − f ( y ) ≤ xy −      =   − ( x − y) ≤               ⇒ f ( x ) = Max { xy − f ( y ) } ≤                ∀x ∈ R   (b )
                       2    2 2            2                              y∈R                           2
                           x2
 ( a ) + (b) ⇒ f ( x) =       . Th l i th y ñúng.
                           2
Ví d 4: Tìm f : R → R th a mãn:

          f ( x + y ) ≥ f ( x ) f ( y ) ≥ 2008x + y ∀x, y ∈ R    ( 4) .
L i gi i:
                                        2
Cho x = y = 0 ⇒ f ( 0 ) ≥ ( f ( 0 ) ) ≥ 1 ⇒ f ( 0 ) = 1 .

Cho
                                                                                                  1
 x = − y ∈ R ⇒ 1 = f ( 0 ) ≥ f ( x ) f ( − x ) ≥ 1⇒ f ( x ) f ( − x ) = 1⇒ f ( x ) =                   ∀x ∈ R   (a) .
                                                                                               f ( −x)

                                     f ( x ) ≥ 2008 x > 0
                                    
Cho y = 0; x ∈ R ⇒ f ( x ) ≥ 2008 ⇒        x
                                                                                (b) .
                                     f ( − x ) ≥ 2008 > 0
                                                       −x
                                    
                                     1        1
Theo ( a ) + ( b ) ⇒ f ( x ) =             ≤        = 2008x       ( c ) . ( b ) + ( c ) ⇒ f ( x ) = 2008x . Th       l i
                                  f ( − x ) 2008− x
th y ñúng.
Ví d 5: Tìm f :        [ a; b ]   → [ a ; b ] th a mãn:

          f ( x ) − f ( y ) ≥ x − y ∀x, y ∈ [ a ; b ] (a < b cho trư c)                 (5).

L i gi i:
Cho x = a ; y = b ⇒ f ( a ) − f ( b ) ≥ a − b = b − a ( a ) .

vì f ( a ) , f ( b )∈ [ a ; b ] nên f ( a ) − f ( b ) ≤ a − b = b − a ( b ) .




                                                        11
 f
                                                  (a) = a
                                            
                                             f   (b) = b
( a ) + ( b ) ⇒ f ( a ) − f ( b ) = b − a ⇔              .
                                             
                                             f   (a) = b
                                             f
                                            
                                                 (b) = a
        f (a) = a
       
+) N u            thì:
        f (b) = b
       
Ch n y = b ; x ∈ [ a ; b ] ⇒ f ( x ) ≤ x ( c ) .

Ch n y = a ; x ∈ [ a ; b ] ⇒ f ( x ) ≥ x ( d ) .

(c) + (d ) ⇒ f ( x) = x .
        f (a) = b
       
+) N u            thì:
        f (b) = a
       
Ch n y = b ; x ∈ [ a ; b ] r i ch n y = a ; x ∈ [ a ; b ] như trên ta ñư c: f ( x ) = a + b − x . Th
l i th y ñúng.
Nh n xét:
      +) T VD1 → VD5 là các BPT hàm. Cách gi i nói chung là tìm các giá tr ñ c bi t – có
th tính ñư c trư c. Sau ñó t o ra các BðT “ngư c nhau” v hàm s c n tìm ñ ñưa ra k t
lu n v hàm s .
     +) Vi c ch n các trư ng h p c a bi n ph i có tính “k th a”. T c là cái ch n sau ph i
d a vào cái ch n trư c nó và th các kh năng có th s d ng k t qu v a có ñư c.
Ví d 6: Tìm f : R → R th a mãn:

                           π 
          f ( 0 ) = a ; f   = b ( a, b cho tr−íc )
                           2                                            (6) .
          f ( x + y ) + f ( x − y ) = 2 f ( x ) cos y ∀x, y ∈ R
         
L i gi i:
            π                        π         π
Cho y =       ; x ∈ R ta ñư c: f  x +  + f  x −  = 0             (a) .
            2                        2         2

Cho x = 0; y ∈ R ta ñư c: f ( y ) + f ( − y ) = 2a cos y           (b) .
            π                    π              π    
Cho x =       ; y ∈ R ta ñư c: f  + y  +      f  − y  = 2b cos y               (c) .
            2                    2              2    




                                                     12
       π            π
                      f    x+ +       f x− =0
                             2            2
                             π          π                  π
( a ) + (b) + ( c ) ⇒  f   x− +       f  − x  = 2a cos  x −  .
                             2          2                  2
                               π     π    
                      f     x +  + f  − x  = 2b cos x
                               2     2    
Gi i h ta ñư c: f ( x ) = a cos x + b sin x . Th l i th y ñúng.

Ví d 7: Tìm f : R → R th a mãn: f ( x ) f ( y ) = f ( x + y ) + sin x sin y ∀x, y ∈ R             (7) .
L i gi i: Ta th y f ( x ) = cos x là m t hàm s th a mãn.

                           2             f (0) = 0
Cho x = y = 0 ⇔ ( f ( 0 ) ) = f ( 0 ) ⇔            .
                                         f (0) = 1
                                        
N u f ( 0 ) = 0 thì: Cho y = 0; x ∈ R ⇒ f ( x ) = − f ( 0 ) = 0 ∀x ∈ R . Th l i ta ñư c:

sin x sin y = 0 ∀x, y ∈ R ⇒ vô lý. V y f ( x ) = 0 không là nghi m (7).

N u f ( 0 ) = 1 thì cho

x = − y ⇒ f ( x ) f ( − x ) = 1 + ( − sin 2 x ) = cos 2 x ⇒ f ( x ) f ( − x ) = cos 2 x ( a ) .

                     π 
          f           =0
       π              2
Cho x = ⇒                     .
       2              π
          f          −  = 0
                      2

      π                 π
N u f   = 0 thì: Cho x = ; y ∈ R th vào (7) suy ra:
      2                 2

      π
f  y +  + sin y = 0 ⇒ f ( y ) = cos y ∀y ∈ R . Th l i th y ñúng.
      2

       π
N u f  −  = 0 tương t như trên ta ñư c: f ( y ) = cos y ∀y ∈ R .
       2
V y hàm s c n tìm là: f ( x ) = cos x .

Ví d 8: Tìm f , g : R → R th a mãn: f ( x ) − f ( y ) = cos ( x + y ) g ( x − y ) ∀x, y ∈ R ( 8) .

L i gi i:
             π                            π                      π    
Ch n x =         − y; y∈ R     (8) ⇒    f  − y  − f ( y) = 0 ⇔ f  − y  = f ( y) (a) .
             2                             2                      2   
             π                       π                             π 
Ch n x =       + y ; y ∈ R ( 8 ) ⇒ f  + y  − f ( y ) = − sin 2 y.g   ( b ) .
             2                       2                             2



                                                         13
π            π                    π 
( a ) + (b) ⇒         f  + y−      f  − y  = − sin 2 y. g   ( c ) .
                        2            2                    2
            π                      π    
Theo (8): f  + y  −              f  − y  = − g (2y) (d ) .
            2                      2    

( c ) + ( d ) ⇒ g ( 2 y ) = sin 2 y. g 
                                           π
                                           ∀y ∈ R ⇒ g ( 2 x ) = a sin 2 x ⇒ g ( x ) = a sin x ∀x ∈ R .
                                         2
           π 
(v i a = g   cho trư c.)
           2
                                                                              a
Cho y = 0; x ∈ R ⇒ f ( x ) − f ( 0 ) = cos x. g ( x ) ⇒ f ( x ) =               sin 2 x + b (b = f ( 0 )) , ∀x ∈ R .
                                                                              2
                            a
                  f ( x ) = sin 2 x + b
Th l i 2 hàm s :            2           (V i a, b là h ng s cho trư c). Th a mãn (8).
                  g ( x ) = a sin x
                 
                                
                                f                       ( − x ) = − f ( x ) ∀x ∈ R ( a )
                                
                                
Ví d 9: Tìm f : R → R th a mãn:  f                      ( x + 1) = f ( x ) + 1 ∀x ∈ R ( b )   .
                                
                                f                        1  f ( x)
                                                          = 2 ∀x ≠ 0 ( c )
                                                         x        x
L i gi i:
           x +1 
Ta tính f        ñ n f ( x ) theo hai cách:
           x 

  x +1       1                      1      f ( x)
f       = f 1 +  = 1 +            f   = 1 + 2 ∀x ≠ 0 ( a ) .
  x          x                      x        x

               x              1 
            f         f 1 −               2
   x +1                      x +1   x +1                              1 
           = 
                x +1
 f                 2
                       =          2
                                       =     1 +                      f −     =
   x   x               x           x                               x +1 
                              
             x +1        x +1 

  x +1
               2
                               1     x +1 
                                                     2
                                                             f ( x + 1) 
=                1 +  − f         =            1 −            =
  x                          x +1    x             ( x + 1) 
                                                                      2
                                                                        
 x +1 
           2
                1+ f ( x) 
             1 −            ∀x ≠ 0, x ≠ 1 ( b ) .
 x               ( x + 1) 
                            2
                             
( a ) + ( b ) ⇒ f ( x ) = x ∀x ≠ 0; x ≠ 1 .
V i x = 0; ( a ) ⇒ f ( 0 ) = 0 th a mãn f ( x ) = x .

V i x = 1; ( a ) ⇒ f ( −1) = − f (1) :

Cho x = 0; ( b ) ⇒ f (1) = 1 ⇒ f ( −1) = −1 th a mãn f ( x ) = x .

                                                                14
V y f ( x ) = x ∀x ∈ R . Th l i th y ñúng .

Ví d 10: Tìm f : R  { 0 } → R th a mãn:

          f (1) = 1 ( a )
         
           1             1 1
         f           = f   . f   ∀x, y ≠ 0 ( b )                                    .
           x+ y          x  y
         ( x + y ) f ( x + y ) = xy f ( x ) f ( y ) ∀x, y tháa m n xy ( x + y ) ≠ 0 ( c )
         
L i gi i:
                                   1      1
Cho x = y ∈ R* , ( b ) ta ñư c: f   = 2 f   ⇒ f ( x ) = 2 f ( 2 x ) ∀x ≠ 0 (*)
                                   2x      x
                                                                 2                         2
Cho x = y ∈ R* , ( c ) ta ñư c: 2 x f ( 2 x ) = x 2 ( f ( x ) ) ⇔ 2 f ( 2 x ) = x ( f ( x ) ) ∀x ≠ 0 (*' ) .
                                                   2
Th (*) vào (*’) suy ra: f ( x ) = x ( f ( x ) )         (* ) .
                                                          "



Gi s : ∃ xo ≠ 1, xo ∈ R* sao cho: f(xo) = 0. Thay x = 1 − xo ; y = xo vào (*”) ta ñư c: f(1) = 0
trái v i gi thi t f(1) = 1. V y f ( x ) ≠ 0 ∀x ≠ 1; x ≠ 0 .

                                                       1
Vì f (1) = 1 ≠ 0 nên t (*”) suy ra f ( x ) =             ∀x ≠ 0 . Th l i th y ñúng.
                                                       x
Ví d 11: Tìm f : R → R th a mãn:

         
         f   (1) = 1 ( a )
         
         
         f   ( x + y ) = f ( x ) + f ( y ) + 2 xy ∀x, y ∈ R ( b ) .
         
         f    1  f ( x)
               = 4 ∀x ≠ 0 ( c )
             x        x
L i gi i:
Cho x = y = 0, ( b ) ⇔ f ( 0 ) = 0

Cho x = y = t ≠ 0, ( b ) ⇔ f ( 2t ) − 2 f ( t ) = 2t 2 (1) .

                 1            1           1 1
Cho x = y =         , (b) ⇔ f   − 2 f       = 2 ( *)
                 2t           t            2t  2t
                 1  f (t )          1  f ( 2t )                       f (t ) f ( 2t )
    (c) ⇒ f 
                                                                                           1
T                 = 4 ;         f  =          4
                                                    . Th vào (*) ta ñư c: 4 − 2       4
                                                                                         = 2             ( 2) .
                t   t              2t  ( 2t )                         t     ( 2t ) 2t
(1) + ( 2 ) ⇒ f ( t ) = t 2   ∀t ≠ 0 . T   f ( 0 ) = 0 ⇒ f ( t ) = t 2 ∀t ∈ R . Th l i th y ñúng.

Ví d 12: Cho hàm s              f : ( 0; + ∞ ) → ( 0; + ∞ ) th a mãn:

            f ( x) 
          f         = y f ( y ) f ( f ( x ) ) ∀x, y ∈( 0; + ∞ ) (12 ) .
            y 

                                                          15
L i gi i: Cho:
 x = y = 1 ⇒ f ( f (1) ) = f (1) . f ( f (1) ) ⇒ f (1) = 1 vì f ( f (1) ) ≠ 0 ⇒ f ( f (1) ) = 1 .

                                                                                      1
                                                                                     f 
                            f (1)                                                    y a .
 x = 1; y ∈ ( 0; + ∞ ) ⇒ f         = y f ( y ) f ( f (1) ) = y f ( y ) ⇔ f ( y ) =       ( )
                            y                                                        y
M t
                         1                                                                              
                         f                                                                       f ( y) 
                                                1 
khác: f ( f ( y ) ) = f     = y f ( y ) f  f    = y f ( y ) f ( y f ( y ) ) = y f ( y )
                             y
                                                                                                   f        
                         y                    y                                                1 
                        
                              
                                                                                                    y 
                                                                                                          

                                            1 1
                               = y f ( y)     f   f ( f ( y )) .
                                            y  y

                                            1 1                      1
Vì f ( f ( y ) ) ≠ 0 nên y f ( y )            f   = 1 ⇔ f ( y)     f   = 1 (b) .
                                            y  y                      y
                               1
( a ) + (b) ⇒ f ( y ) =           ∀y ∈( 0; + ∞ ) . Th l i th y ñúng.
                                y
Ví d 13: Tìm f : R → R th a mãn:

                   1
           f ( 0) = ( a )
                   2                                                                        .
          ∃ a ∈ R : f ( a − y ) f ( x ) + f ( a − x ) f ( y ) = f ( x + y ) ∀x, y ∈ R ( b )
          
L i gi i:
                                         1
Cho x = y = 0, ( b ) ⇒ f ( a ) =           .
                                         2
Cho y = 0; x ∈ R ta ñư c: f ( x ) = f ( x ) . f ( a ) + f ( 0 ) . f ( a − x ) ⇒ f ( x ) = f ( a − x ) ( c ) .
                                                            2               2
Cho y = a − x ; x ∈ R ta ñư c: f ( a ) = ( f ( x ) ) + ( f ( a − x ) )          (d ) .
                                                1
                               2    1  f ( x) = 2
( c ) + ( d ) ⇒ 2 ( f ( x ))       = ⇔              .
                                    2  f ( x) = − 1
                                      
                                                  2
                                               1
N u ∃ xo ∈ R sao cho: f ( xo ) = −               thì:
                                               2
                                                                                 2
 1           (b)
                 x x           x           x  (c)   x  
− = f ( xo ) = f  o + o  = 2 f  o  . f  a − o  = 2  f  o   ≥ 0 ⇒ Vô lí.
 2                2 2           2           2        2 
                 1
V y f ( x) =       ∀x ∈ R . Th l i th y ñúng.
                 2

                                                           16
Ví d 14: (VMO.1995)

Tìm f : R → R th a mãn: f           (( x − y ) ) = x
                                                 2     2                            2
                                                           − 2 y f ( x ) + ( f ( y ) ) ∀x, y ∈ R (14 ) .

L i gi i:

                                     2   f ( 0) = 0
Cho x = y = 0 ⇒ f ( 0 ) = ( f ( 0 ) ) ⇔             .
                                         f ( 0) = 1
                                        
                      y = 0
N u f ( 0 ) = 0 : Cho       ta ñư c: f ( x 2 ) = x 2 ⇒ f ( t ) = t ∀t ≥ 0
                      x ∈ R
                                                                            2                  2
Cho x = y ∈ R ta ñư c:               f ( 0) = x2 − 2 x f ( x ) + ( f ( x ) ) ⇔ ( f ( x ) − x ) = 0 ⇔ f ( x ) = x .
Th l i th y ñúng.
                     y = 0
N u f ( 0 ) = 1: Cho       ta ñư c: f ( x 2 ) = x 2 + 1 ⇔ f ( t ) = t + 1 ∀t ≥ 0 .
                      x∈R
                                                                 2              2
Cho x = 0; y ∈ R ta ñư c: f ( y 2 ) = −2 y + ( f ( y ) ) ⇒ ( f ( y ) ) = f ( y 2 ) + 2 y

                                                                     2   f ( y) = y +1
                                           = y 2 + 1 + 2 y = ( y + 1) ⇒                  .
                                                                         f ( y) = − y −1
                                                                        
Gi s ∃ yo ∈ R sao cho: f ( yo ) = − yo − 1 . Ch n x = y = yo ta ñư c:

                                     2   f ( yo ) = yo − 1
1 = yo − 2 yo f ( yo ) + ( f ( yo ) ) ⇔ 
     2
                                                            .
                                         f ( yo ) = yo + 1
                                        
N u f ( yo ) = yo − 1 ⇒ − yo − 1 = yo − 1 ⇒ yo = 0 v f ( 0 ) = −1 (lo¹i) .

N u f ( yo ) = yo + 1 ⇒ − yo − 1 = yo + 1 ⇒ yo = −1 ⇒ f ( −1) = 0 .

Th a mãn: f ( yo ) = yo + 1 . V y f ( y ) = y + 1 ∀y ∈ R . Th l i th y ñúng.

Ví d 15: (VMO.2005)
Tìm f : R → R th a mãn: f ( f ( x − y ) ) = f ( x ) f ( y ) − f ( x ) + f ( y ) − xy ∀x, y ∈ R (15 ) .

L i gi i:
                                             2
Cho x = y = 0 ⇒ f ( f ( 0 ) ) = ( f ( 0 ) ) . ð t f ( 0 ) = a ⇒ f ( a ) = a 2 .
                               2                                 2
Cho x = y ∈ R ⇒ ( f ( x ) ) = x 2 + f ( a ) ⇒ ( f ( x ) ) = x 2 + a 2 (*) .

             2              2   f ( x) = f (−x)
 ⇒ ( f ( x )) = ( f ( − x )) ⇒                    .
                                f ( x) = − f (−x)
                               
N u ∃ xo ∈ R* sao cho f ( xo ) = f ( − xo ) :

+ Ch n x = 0; y = − xo ⇒ f ( f ( xo ) ) = a f ( − xo ) − a + f ( − xo ) ( a ) .


                                                           17
+ Ch n y = 0; x = − xo ⇒ f ( f ( xo ) ) = a f ( xo ) + a − f ( xo ) ( b ) .

( a ) + ( b ) ⇒ a ( f ( xo ) − f ( − xo ) ) − ( f ( xo ) + f ( − xo ) ) + 2a = 0 ( c ) .
                                                  (*)             2
Vì f ( xo ) = f ( − xo ) nên f ( xo ) = a ⇒ ( f ( xo ) ) = x0 + a 2 ⇒ a 2 = x0 + a 2 ⇒ xo = 0 trái v i
                                                            2                2



gi thi t xo ∈ R* .

V y f ( x ) = − f ( − x ) ∀x ∈ R . Ta th y (c) không ph thu c vào xo nên ta có:

a ( f ( x ) − f ( − x ) ) − ( f ( x ) + f ( − x ) ) + 2a = 0 ( c ) . Thay f ( x ) = − f ( − x ) suy ra:

                       a = 0
a ( f ( x ) + 1) = 0 ⇔               .
                        f ( x ) = −1
            (*)         2        f ( x) = x
+ N u a = 0 ⇒ ( f ( x )) = x2 ⇔              .
                                 f ( x) = −x
                                
Gi s t n t i xo ∈ R* ñ             f ( xo ) = xo . Khi ñó (b) suy ra:

xo = f ( xo ) = a xo + a − xo ⇒ xo = 0 trái gi thi t xo ∈ R* .

V y f ( x ) = − x ∀x ∈ R . Th l i th y ñúng

+ N u f ( x ) = −1 ∀x ∈ R . Th l i ta ñư c (15 ) ⇔ xy = 2 ∀x, y ∈ R . Vô lí.

V y hàm s c n tìm là: f ( x ) = − x .

Nh n xét: Có m t suy lu n hay nh m l n ñư c s d ng các VD:
                                1 
                 2 1   f ( x) = 2                       2          2   f ( y ) = y +1 
VD13  ( f ( x ) ) = ⇔                ; VD14  ( f ( y ) ) = ( y + 1) ⇔                   ;
                   4   f ( x) = − 1                                    f ( y ) = − y − 1
                                                                                         
                      
                                   2

                 2         f ( x) = x 
VD15  ( f ( x ) ) = x 2 ⇔               , ñó là hi u sai:
                           f ( x) = −x 
                                       

                                  1
             2     1    f ( x ) = 2 ∀x ∈ R
( f ( x ))       =   ⇔                       ;
                   4    f ( x ) = − 1 ∀x ∈ R
                       
                                    2

             2           2      f ( y ) = y + 1 ∀x ∈ R
( f ( y ) ) = ( y + 1)       ⇔                           ;
                                f ( y ) = − y − 1 ∀x ∈ R
                               
             2           f ( x ) = x ∀x ∈ R
( f ( x ))       = x2 ⇔                       .
                         f ( x ) = − x ∀x ∈ R
                        



                                                             18
2    1
Th c t thư ng là như v y nhưng v m t logic thì không ñúng. ( f ( x ) ) =                            thì f ( x ) có th
                                                                                                  4
                              1                                                                         1
                               2 ( x ≥ 0)
                                                                                      2    1   f ( x) = 2
là hàm khác n a như f ( x ) =               . Như v y                    ( f ( x ))       = ⇔               ch
                              − 1 ( x < 0 )                                                4   f ( x) = − 1
                               2
                                                                                              
                                                                                                           2
                                                                                                        1
ñúng v i m i x c th ch không th k t lu n ch có hai hàm s                                     f ( x) =     ∀x ∈ R ho c
                                                                                                        2
               1
 f ( x) = −      ∀x ∈ R .
               2
                                                                   1                        1
ð gi i quy t v n ñ này ta thư ng “th ” f ( x ) =                     ∀x ∈ R ho c f ( x ) = − ∀x ∈ R vào ñ
                                                                   2                        2
                                                                               1
bài ñ tìm hàm s không th a mãn (trong VD13 thì f ( x ) =                         không th a mãn) sau ñó l p
                                                                               2
                                           1
lu n ph ñ nh là ∃ xo : f ( xo ) = −          ñ d n ñ n vô lí!
                                           2
Ví d 16: Tìm f : (0,1) → ℝ th a mãn: f(xyz) = xf(x) + yf(y) +zf(z) ∀x, y , z ∈ (0,1) .
L i gi i:
Ch n x = y = z: f(x3) = 3xf(x).
Thay x, y, z b i x2: f(x6) = 3 x2 f(x2).
M t khác: f(x6) = f(x. x2 .x3) = xf(x) + x2 f(x2) + x3 f(x3).
⇒ 3 x2 f(x2) = xf(x) + x2 f(x2) + 3x4 f(x) ⇔ 2 x2 f(x2) = xf(x) + 3x4 f(x)
                 3x3 + 1
⇒ f ( x2 ) =             f ( x), ∀x ∈ ℝ
                   2
Thay x b i x3 ta ñư c :
          3 x9 + 1
f ( x6 ) =           f ( x 3 ), ∀x ∈ ℝ
               2
                    3x9 + 1
⇒ 3x 2 f ( x 2 ) =             3 xf ( x), ∀x ∈ ℝ
                        2
       3x3 + 1                3x9 + 1
⇒ 3x 2             f ( x) =             3 xf ( x), ∀x ∈ ℝ
           2                      2
⇒ f ( x) = 0, ∀x ≠ 0
  V y f(x) = 0 v i m i x ∈(0; 1).
                                                       BÀI T P
                                                                 5
1) Tìm f : N → R th a mãn: f ( 0 ) ≠ 0; f (1) =                    ;
                                                                 2
             f ( x ) f ( y ) = f ( x + y ) + f ( x − y ) ∀x, y ∈ N , x ≥ y .

2) Tìm f : N → R th a mãn: f ( m + n ) + f ( n − m ) = f ( 3n ) ∀m, n ∈ N , n ≥ m .

                                                            19
3) Tìm f : R → R th a mãn: f ( x f ( y ) ) = y f ( x ) x, y ∈ R .

4) Tìm f : R → R th a mãn: f              ( ( x + 1) f ( y ) ) = y ( f ( x ) + 1)    x, y ∈ R .

5) Tìm f : ( 0; + ∞ ) → ( 0; + ∞ ) th a mãn:

 f ( x ) = Max  x 2 y + y 2 x − f ( y )  ∀x ∈ ( 0; + ∞ ) .
          y∈( 0; +∞ )                   

6) Tìm f : R → R th a mãn: f ( xy ) − f ( x − y ) + f ( x + y + 1) = xy + 2 x + 1 ∀x, y ∈ R .

                                              f ( xy ) = f ( x ) f ( y )
                                             
7) Tìm f : [ 1; + ∞ ) → [ 1; + ∞ ) th a mãn:                             ∀x, y ∈ [ 1; + ∞ ) .
                                              f ( f ( x )) = x
                                             
8) Tìm f : R → R th a mãn: f ( xy ) = f ( x ) f ( y ) − f ( x + y ) + 1 ∀x, y ∈ R .

9) Tìm f : R → R th a mãn:

              ( f ( x ) + f ( z ) ) ( f ( y ) + f ( t ) ) = f ( xy − zt ) + f ( xt + zy )   ∀x, y, z , t ∈ R .

10) Tìm f : R → R th a mãn: f ( x 2 − y 2 ) = x f ( y ) − y f ( x ) ∀x, y ∈ R .

11) Tìm f : N → [ 0; + ∞ ) th a mãn:

                                                          1
              f (1) = 1; f ( m + n ) + f ( m − n ) =
                                                          2
                                                            ( f ( 2m ) + f ( 2n ) ) ∀m, n ∈ N , m ≥ n .
                               x + y  f ( x) + f ( y)
12) Tìm f : Z → R th a mãn: f        =                ∀x, y ∈ Z ; ( x + y )⋮ 3 .
                               3             2

13) Tìm f : N → N th a mãn: 3 f ( n ) − 2 f ( f ( n ) ) = n ∀n ∈ N .

14) Tìm f : Z → Z th a mãn:

          f (1) = a ∈ Z ; f ( m + n ) + f ( m − n ) = 2 ( f ( m ) + f ( n ) ) ∀m, n ∈ Z .

15) Tìm f : R → R th a mãn: f ( x 3 + 2 y ) = f ( x + y ) + f ( 3 x + y ) + 1 ∀x, y ∈ R .

16) Tìm f : R → R th a mãn: x 2 f ( x ) + f (1 − x ) = 2 x − x 4                    ∀x ∈ R .

Phương pháp 4: S d ng tính ch t nghi m c a m t ña th c.
Ví d 1: Tìm P(x) v i h s th c, th a mãn ñ ng th c:
( x3 + 3 x 2 + 3 x + 2) P( x − 1) = ( x 3 − 3 x 2 + 3 x − 2) P( x), ∀x (1)
L i gi i:
(1) ⇔ ( x + 2)( x 2 + x + 1) P ( x − 1) = ( x − 2)( x 2 − x + 1) P( x), ∀x
Ch n:              x = −2 ⇒ P ( −2) = 0
                    x = −1 ⇒ P(−1) = 0
                    x = 0 ⇒ P(0) = 0
                    x = 1 ⇒ P(1) = 0
  V y: P(x) = x(x – 1)(x + 1)(x + 2)G(x).
                                                            20
Thay P(x) vào (1) ta ñư c:
  ( x + 2)( x 2 + x + 1)( x − 1)( x − 2) x( x + 1)G ( x − 1) = ( x − 2)( x 2 − x + 1) x( x − 1)( x + 1)( x + 2)G ( x), ∀x

  ⇒ ( x 2 + x + 1) G ( x − 1) = ( x 2 − x + 1)G ( x), ∀x
     G ( x − 1)       G ( x)
  ⇔   2
                 = 2          , ∀x
    x − x +1 x + x +1
           G ( x − 1)            G ( x)
  ⇔         2
                             = 2        , ∀x
    ( x − 1) + ( x − 1) + 1 x + x + 1
                   G ( x)
ð t     R( x) =     2
                                   (x ≠ 0, ± 1, -2)
                  x + x +1
    ⇒ R ( x) = R ( x − 1) (x ≠ 0, ± 1, -2)
    ⇒ R( x) = C

V y P( x) = C ( x 2 + x + 1) x( x − 1)( x + 1)( x + 2)
Th l i th y P(x) th a mãn ñi u ki n bài toán.
Chú ý: N u ta xét P(x) = (x3 + 1)(x – 1) thì P(x + 1) = (x3 + 3x2 + 3x + 2)x.
Do ñó (x3 + 3x2 + 3x + 2)xP(x) = (x2 – 1)(x2 – x + 1)P(x + 1). T ñó ta có bài toán sau:
Ví d 2: Tìm ña th c P(x) v i h s th c, th a mãn ñ ng th c:
(x3 + 3x2 + 3x + 2)xP(x) = (x2 – 1)(x2 – x + 1)P(x + 1) v i m i x.
Gi i quy t ví d này hoàn toàn không có gì khác so v i ví d 1.
Tương t như trên n u ta xét: P(x) = (x2 + 1)(x2 – 3x + 2) thì ta s có bài toán sau:
Ví d 3: Tìm ña th c P(x) v i h s th c th a mãn ñ ng th c:
(4 x 2 + 4 x + 2)(4 x 2 − 2 x ) P ( x) = ( x 2 + 1)( x 2 − 3 x + 2) P(2 x + 1), ∀x ∈ ℝ
Các b n có th theo phương pháp này mà t sáng tác ra các ñ toán cho riêng mình.
Phương pháp 5: S d ng phương pháp sai phân ñ gi i phương trình hàm.
1. ð nh nghĩa sai phân:
Xét hàm x(n) = xn:
Sai phân c p 1 c a hàm xn là: △ xn = xn+1 − xn
Sai phân câp 2 c a hàm xn là: △2 xn =△ xn +1 −△ xn = xn + 2 − 2 xn +1 + xn
                                                    k
Sai phân câp k c a hàm xn là: △k xn = ∑ (−1)i Cki xn + k −i
                                                   i =0

2. Các tính ch t c a sai phân:
+) Sai phân các c p ñ u ñư c bi u th qua các giá tr hàm s .
+) Sai phân có tính tuy n tính: ∆ k (af + bg ) = a∆ k f + b∆ k g
+) N u xn ña th c b c m thì ∆ k xn :
         Là ña th c b c m – k n u m > k.
         Là h ng s n u m = k.
        Là 0 n u m < k.

                                                           21
3. N i dung c a phương pháp này là chuy n bài toán phương trình hàm sang bài toán dãy
s và dùng các ki n th c dãy s ñ tìm ra các hàm s c n tìm.
Ví d 1: Tìm f: N → R tho mãn : f(1) = 1 và
     2f(n).f(n+k) = 2f(k-n) + 3f(n).f(k) ∀ k, n∈N, k≥ n.
L i gi i:
                                              f (0) = 0
Cho n = k = 0 ta ñư c: (f(0))2 + 2f(0) = 0 ⇔             .
                                              f (0) = −2
     + N u f(0) = 0 thì ch n n = 0, k∈ N ta ñư c: f(k) = 0 trái gi thi t f(1) = 1.
     + N u f(0) = - 2 thì ch n n = 1, k∈ N* ta ñư c: 2.f(k+1) - 3.f(k) - 2.f(k-1) = 0.

                              u0 = −2; u1 = 1
                              
ð t uk = f(k) ta ñư c dãy s :                                  *
                                                                  .
                              2uk +1 − 3uk − 2uk −1 = 0 ∀k ∈ ℕ
                              
                                1
T ñây tìm ñư c uk = f(k) = −2.(− ) k ∀k ∈ N . Th l i th y ñúng.
                                2
Ví d 2 (D tuy n IMO 1992): Cho a, b> 0. Tìm f: [0; +∞) → [0; +∞) tho mãn :
                                       f(f(x))+a.f(x) = b.(a+b).x ∀x∈ [0; +∞) (2)
L i gi i:
C ñ nh x∈ [0; +∞) và ñ t u0 = x, u1 = f(x), un+1 = f(un). T (2) ta ñư c :
un+2 + a.un+1 - b.(a + b).un = 0. Gi i dãy s trên ta ñư c: un = c1.bn + c2.(-a -b)n (*).
                                                     un             b n                                b
Vì un ≥ 0 ∀n∈N nên ta có: 0 ≤                             n
                                                            = c1.(     ) + c2 .(−1) n . M t khác: 0 <     < 1 nên
                                                 ( a + b)          a+b                                a+b
          b n                                                         un
 lim (       ) = 0 . Do ñó, n u c2 > 0 thì khi n l và n ñ l n thì            < 0 vô lí !; còn n u
n →+∞    a+b                                                      ( a + b) n
                                                            un
c2 < 0 thì khi n ch n và n ñ l n thì                               < 0 vô lí !. V y c2 = 0. Thay vào (*) ta ñư c
                                                        ( a + b) n
un = c1.bn. T u0 = x suy ra c1 = x và f(x) = bx. Do ñó f(x) = bx ∀x∈[0;+∞). Th l i th y ñúng.
 Ví d 3: Tìm t t c các hàm f : ℝ → ℝ th a mãn:
                  f(f(x)) = 3f(x) – 2x , ∀x ∈ ℝ
L i gi i :
Thay x b i f(x) ta ñư c:
f(f(f(x))) = 3f(f(x)) – 2f(x) , ∀x ∈ ℝ
    ………………………..
 f (... f ( x)) = 3 f (... f ( x)) − 2 f (... f ( x))
     n+ 2                n +1                n

Hay f n+ 2 ( x) = 3 f n+1 ( x) − 2 f n ( x), n ≥ 0
ð t xn = f n ( x ), n ≥ 0 ta ñư c phương trình sai phân: xn+ 2 = 3 xn +1 − 2 xn
Phương trình ñ c trưng là: λ 2 − 3λ + 2 = 0 ⇔ λ = 1 ∨ λ = 2

                                                              22
V y xn = c1 + c2 2 n

        x0 = c1 + c2 = x
Ta có: 
        x1 = c1 + 2c2 = f ( x )
T ñó ta ñư c c1 = 2 x − f ( x), c2 = f ( x) − x
V y f ( x) = x + c2 ho c f ( x) = 2 x − c1

Phương pháp 6: Phương pháp s d ng ánh x .
Ví d 1: Tìm f: N*→ N* tho mãn:
       f(f(n)+m) = n+f(m+2007) ∀ m, n∈N* (1).
L i gi i:
Trư c h t ta ch ng minh f là ñơn ánh.
Th t v y: f(n1) = f(n2) ⇒ f(f(n1)+1) = f(f(n2)+1) ⇒ n1 + f(1+2007) = n2 + f(1+2007) ⇒
n1 = n2. V y f là ñơn ánh.
M t khác t (1) suy ra: ∀ m, n ∈ N*, f(f(n) + f(1)) = n + f(f(1) + 2007) ⇒ f(f(n)+f(1)) = n + 1
+ f(2007+2007) = f(f(n+1)+2007). Vì f là ñơn ánh nên ta có:       f(n) + f(1) = f(n+1) + 2007 ⇒
f(n+1) - f(n) = f(1) - 2007. ð t f(1) - 2007 = a. Khi ñó ta có f(n) = n.a + 2007. Thay l i (10) ta
ñư c a2n = n ∀n∈N* ⇒ a2 = 1 ⇒ a = 1 ⇒ f(n) = n+2007.
Ví d 2: Tìm f: R → R tho mãn: f(xf(x)+f(y)) = (f(x))2+y ∀ x, y∈R (2).
L i gi i:
D ràng ch ng minh f là ñơn ánh.
M t khác, c ñ nh x thì ∀t∈R t n t i y = t - (f(x))2 ñ f(xf(x) + f(y)) = t. V y f là toàn ánh,
do ñó f là song ánh. Suy ra t n t i duy nh t a∈R sao cho f(a) = 0.
Cho x = y = a ta ñư c f(0) = a.
Cho x = 0, y = a ta ñư c f(0) = a2 + a. V y a = a2 + a hay a = 0 ⇒ f(0) = 0.
Cho x = 0, y∈R ta ñư c f(f(y)) = y (a).
Cho y = 0, x∈R ta ñư c f(x.f(x)) = (f(x))2 ⇒ f(f(x).f(f(x))) = (f(f(x)))2. Theo (a) ta ñư c
f(f(x).x)) = x2 ⇒ (f(x))2 = x2 ⇒ f(x) = x ho c f(x) = -x.
Gi s t n t i a, b∈R* ñ f(a) = a, f(b) = -b. Khi ñó thay x = a, y = b thì t (2) suy ra: f(a2 - b)
= a2 + b. Mà (a2 + b)2 ≠ (a2 - b)2 v i a, b∈ R* trái v i kh ng ñ nh (f(x))2 = x2. V y có hai hàm
s là f(x) = x, ∀x∈R ho c f(x) = -x ∀x∈R. Th l i th y ñúng.
Phương pháp 7: phương pháp ñi m b t ñ ng.
1. ð c trưng c a hàm:
       Như ta ñã bi t, phương trình hàm là m t phương trình thông thư ng mà nghi m c a
nó là hàm. ð gi i quy t t t v n ñ này, c n phân bi t tính ch t hàm v i ñ c trưng hàm.
Nh ng tính ch t quan tr c ñư c t ñ i s sang hàm s , ñư c g i là nh ng ñ c trưng hàm.
+) Hàm tuy n tính f(x) = ax , khi ñó f(x + y) = f(x) + f(y). V y ñ c trưng hàm tuy n tính là:
f(x + y) = f(x) + f(y) v i m i x, y.
                                                  23
x+ y
+) Hàm b c nh t f(x) = ax + b, khi ñó f(x) + f(y) = 2 f (          ) . V y ñ c trưng hàm     ñây là
                                                               2
   x + y  f ( x) + f ( y )
 f       =                 , ∀x, y ∈ ℝ
   2             2
        ð n ñây thì ta có th               nêu ra câu h i là: Nh ng hàm nào có tính ch t
 f ( x + y ) = f ( x ) + f ( y ), ∀x, y ∈ ℝ . Gi i quy t v n ñ ñó chính là d n ñ n phương trình hàm.
V y phương trình hàm là phương trình sinh b i ñ c trưng hàm cho trư c.
+) Hàm lũy th a f ( x) = x k , x > 0 ð c trưng là f(xy) = f(x)f(y).
+) Hàm mũ f ( x) = a x (a > 0, a ≠ 1) ð c trưng hàm là f(x + y) = f(x)f(y), ∀x, y ∈ ℝ
+) Hàm Lôgarit f ( x) = log a x (a>0,a ≠ 1) ð c trưng hàm là f(xy) = f(x) + f(y).
+) f(x) = cosx có ñ c trưng hàm là f(x + y) + f(x – y) = 2f(x)f(y).
  Hoàn toàn tương t ta có th tìm ñư c các ñ c trưng hàm c a các hàm s f(x) =sinx,
f(x) = tanx và v i các hàm Hypebolic:
                          e x − e− x
+) Sin hypebolic shx =
                              2
                          e x + e− x
+) cos hypebolic chx =
                              2
                         shx e x − e − x
+) tan hypebolic thx =      =
                         chx e x + e − x
                             chx e x + e − x
+) cot hypebolic cothx =        =
                             shx e x − e − x
+) shx có TXð là ℝ t p giá tr là ℝ
  chx có TXð là ℝ t p giá tr là [1, +∞ )
  thx có TXð là ℝ t p giá tr là (-1,1)
  cothx có TXð là ℝ  {0} t p giá tr là (−∞, −1) ∪ (1, +∞ )
Ngoài ra b n ñ c có th xem thêm các công th c liên h gi a các hàm hypebolic, ñ th c a
các hàm hypebolic.
2. ði m b t ñ ng:
       Trong s h c, gi i tích, các khái ni m v ñi m b t ñ ng, ñi m c ñ nh r t quan tr ng
và nó ñư c trình bày r t ch t ch thông qua m t h th ng lý thuy t. ñây, tôi ch nêu ng
d ng c a nó qua m t s bài toán v phương trình hàm.
Ví d 1: Xác ñ nh các hàm f(x) sao cho: f(x+1) = f(x) + 2 ∀x ∈ ℝ.
L i gi i:
Ta suy nghĩ như sau: T gi thi t ta suy ra c = c + 2 do ñó c = ∞
Vì v y ta coi 2 như là f(1) ta ñư c f(x + 1) = f(x) + f(1) (*)
Như v y ta ñã chuy n phép c ng ra phép c ng. D a vào ñ c trưng hàm, ta ph i tìm a:
f(x) = ax ñ kh s 2. Ta ñư c (*) ⇔ a ( x + 1) = ax + 2 ⇔ a = 2
V y ta làm như sau: ð t f(x) = 2x + g(x). Thay vào (*) ta ñư c:

                                                  24
2(x + 1) + g(x + 1) = 2x + g(x) + 2, ∀x ∈ ℝ
ði u này tương ñương v i g(x + 1) = g(x), ∀x ∈ ℝ V y g(x) là hàm tu n hoàn v i chu kì 1.
ðáp s f(x) = 2x + g(x) v i g(x) là hàm tu n hoàn v i chu kì 1.
Nh n xét: Qua ví d 1, ta có th t ng quát ví d này, là tìm hàm f(x) th a mãn:
        f(x + a) = f(x) + b, ∀x ∈ ℝ , a, b tùy ý.
Ví d 2: Tìm hàm f(x) sao cho: f(x + 1) = - f(x) + 2, ∀x ∈ ℝ (2).
L i gi i:
ta cũng ñưa ñ n c = -c + 2 do ñó c = 1.
v y ñ t f(x) = 1 + g(x), thay vào (2) ta ñư c phương trình: g(x + 1) = - g(x), ∀x ∈ ℝ
                                                1
              g ( x + 1) = − g ( x)   g ( x ) = [ g ( x) − g ( x + 1)]
Do ñó ta có:                        ⇔          2                       ∀x ∈ ℝ (3).
              g ( x + 2) = g ( x)     g ( x + 2) = g ( x)
                                      
                                                             1
Ta ch ng minh m i nghi m c a (3) có d ng: g ( x) =             [ h( x) − h( x + 1) ] , ∀x ∈ ℝ   ñó h(x) là
                                                             2
hàm tu n hoàn v i chu kì 2.
Nh n xét: Qua ví d này, ta có th t ng quát thành: f(x + a) = - f(x) + b, ∀x ∈ ℝ , a, b tùy ý.
Ví d 3: Tìm hàm f(x) th a mãn: f(x + 1) = 3f(x) + 2, ∀x ∈ ℝ (3).
Gi i:
Ta ñi tìm c sao cho c = 3c + 2 d th y c = -1. ð t f(x) = -1 + g(x). Lúc ñó (3) có d ng:
g(x + 1) = 3g(x) ∀x ∈ ℝ
Coi 3 như g(1) ta ñư c: g(x + 1) = g(1).g(x) ∀x ∈ ℝ (*).
T ñ c trưng hàm, chuy n phép c ng v phép nhân, ta th y ph i s d ng hàm mũ:
a x +1 = 3a x ⇔ a = 3
V y ta ñ t: g ( x) = 3x h( x) thay vào (*) ta ñư c: h(x + 1) = h(x) ∀x ∈ ℝ .
V y h(x) là hàm tu n hoàn chu kì 1.
K t lu n: f ( x) = −1 + 3x h( x) v i h(x) là hàm tu n hoàn chu kì 1.
Nh n xét: ví d 3 này, phương trình t ng quát c a lo i này là: f(x + a) = bf(x) + c ∀x ∈ ℝ ;
a, b, c tùy ý.
+) V i 0< b ≠ 1: chuy n v hàm tu n hoàn.
+) V i 0< b ≠ 1: chuy n v hàm ph n tu n hoàn.
Ví d 4: Tìm hàm f(x) th a mãn f(2x + 1) = 3f(x) – 2 ∀x ∈ ℝ (4)
Gi i:
Ta có: c = 3c – 2 suy ra c = 1. ð t f(x) = 1 + g(x). Khi ñó (4) có d ng:
g(2x + 1) = 3g(x) ∀x ∈ ℝ (*)
Khi bi u th c bên trong có nghi m ≠ ∞ thì ta ph i x lý cách khác.
T 2x + 1 = x suy ra x = 1. V y ñ t x = -1 + t ta có 2x + 1 = -1 + 2t. Khi ñó (*) có d ng:
g(-1 + 2t) = 3g(-1 + t ) ∀t ∈ ℝ .
ð t h(t) = g(-1 + 2t), ta ñư c h(2t) = 3h(t) (**). Xét 2t = t ⇔ t = 0 , (2t ) m = 3.t m ⇔ m = log 2 3
Xét ba kh năng sau:
                                                    25
+) N u t = 0 ta có h(0) = 0.
+) N u t> 0 ñ t h(t ) = t log2 3ϕ (t ) thay vào (3) ta có: ϕ (2t ) = ϕ (t ), ∀t > 0 . ð n ñây ta ñưa v ví
d hàm tu n hoàn nhân tính.
                                                                  ϕ (2t ) = −ϕ (t ), ∀t < 0
+) N u t < 0 ñ t h(t ) =| t |log2 3 ϕ (t ) thay vào (3) ta ñư c ⇔ 
                                                                  ϕ (4t ) = ϕ (t ), ∀t < 0
          1
 ϕ (t ) = [ϕ (t ) − ϕ (2t )] , ∀t < 0
⇔         2                           .
 ϕ (4t ) = ϕ (t ), ∀t < 0
 
Nh n xét: Bài toán t ng quát c a d ng này như sau: f (α x + β ) = f (ax ) + b α ≠ 0, ± 1 . Khi
ñó t phương trình α x + β = x ta chuy n ñi m b t ñ ng v 0, thì ta ñư c hàm tu n hoàn nhân
tính.
+) N u a = 0 bài toán bình thư ng.
+) N u a = 1 ch ng h n xét bài toán sau: “Tìm f(x) sao cho f(2x + 1) = f(x) – 2, ∀x ≠ -1 (1)”.
Xét: 2x + 1 = x ⇔ x = −1 nên ñ t x = -1 + t thay vào (1) ta ñư c: f(-1 + 2t) = f(-1 + t) + 2,
∀t ≠ 0 . ð t g(t) = f( - 1 + t) ta ñư c: g(2t) = g(t) + 2 ∀t ≠ 0 (2). T tích chuy n thành t ng
nên là hàm logarit.
                                           1
Ta có log a (2t ) = log a t − 2 ⇔ a =         . V y ñ t g (t ) = log   1    t + h(t ) . Thay vào (2) ta có
                                            2                           2

h(2t ) = h(t ), ∀t ≠ 0 . ð n ñây bài toán tr nên ñơn gi n.

Phương pháp 8: phương pháp s d ng h ñ m.
Ta quy ư c ghi m = (bibi-1...b1)k nghĩa là trong h ñ m cơ s k thì m b ng bibi-1...b1.
Ví d 1 (Trích IMO năm 1988):
        Tìm f: N*→ N* tho mãn: f(1) = 1, f(3) = 3, f(2n) = f(n),
        f(4n+1) = 2f(2n+1) - f(n); f(4n+3) = 3f(2n+1) - 2f(n) ∀n∈N* (12).
L i gi i:
Tính m t s giá tr c a hàm s và chuy n sang cơ s 2 ta có th d ñoán ñư c:
“∀n∈N*, n = (bibi-1...b1)2 thì f(n) = (b1b2 ...bi)2” (*). Ta s ch ng minh (*) b ng quy n p.
+ V i n = 1, 2, 3, 4 d ki m tra (*) là ñúng.
+ Gi s (*) ñúng cho k < n, ta s ch ng minh (*) ñúng cho n (v i n ≥ 4). Th t v y, ta xét các
kh năng sau:
• N u n ch n, n = 2m. Gi s m = (bibi-1...b1)2, khi ñó n = 2m = (bibi-1...b10)2 ⇒
f(n) = f((bibi-1...b10)2) = f(2m) = f(m) = f((bibi-1...b1)2) = (b1b2 ...bi)2 = (0b1b2 ...bi)2 ⇒ (*)
ñúng.
• N u n l và n = 4m + 1. Gi s m = (bibi-1...b1)2, khi ñó n = (bibi-1...b101)2 ⇒
f(n) = f((bibi-1...b101)2) = f(4m+1) = 2.f(2m+1) - f(m) = 2.f((bibi-1...b11)2) - f((bibi-1...b1)2) =
(10)2.(1b1b2 ...bi)2 - ( b1b2 ...bi)2 = (1b1b2 ...bi0)2 - ( b1b2 ...bi)2 = (10b1b2 ...bi)2 ⇒ (*) ñúng.

                                                    26
• N u n l và n = 4m + 3. Gi s m = (bibi-1...b1)2, khi ñó n = (bibi-1...b111)2 ⇒
f(n) = f((bibi-1...b111)2) = f(4m+3) = 3f(2m+1) - 2f(m) = 3f((bibi-1...b11)2) - 2f((bibi-1...b1)2) =
(11)2.(1b1b2 ...bi)2 - (10)2.(b1b2 ...bi)2 = (11b1b2 ...bi)2 ⇒ (*) ñúng.
V y (*) ñúng và hàm f ñư c xác ñ nh như (*).
Ví d 2 (Trích ñ thi c a Trung Qu c):
          Tìm hàm s f: N* → N* th a mãn:
          1) f(1) =1;
          2) f(2n) < 6f(n);
          3) 3f(n)f(2n+1) = f(2n)(3f(n)+1) ∀n∈N*.
L i gi i:
Vì f(n)∈N* nên (3f(n), 3f(n)+1) = 1. T            3) suy ra 3f(n) | f(2n). K t h p v i 2) suy ra
                                              *
f(2n) = 3f(n) và f(2n+1) = 3f(n)+1 ∀n∈N .
Th m t s giá tr ta th y f(n) ñư c xác ñ nh như sau:
“V i n = (b1b2…bi)2 thì f(n) = (b1b2…bi)3 ∀n∈N*” (*). Ta ch ng minh (*) b ng quy n p.
+ V i n = 1, 2, 3, 4 thì hi n nhiên (*) ñúng.
+ Gi s (*) ñúng cho k < n (v i n ≥ 4). Ta ch ng minh (*) ñúng cho n.
• N u n ch n: n = 2m. Gi s m = (c1c2…cj)2 thì n = 2m = (c1c2…cj0)2. Khi ñó:
f(n) = f(2m) = 3f(m) = 3.f((c1c2…cj)2) = (10)3.(c1c2…cj)3 = (c1c2…cj0)3 ⇒ (*) ñúng cho n
ch n.
• N u n l : n = 2m + 1 ⇒ n = (c1c2…cj1)2. Khi ñó:
f(n) = f(2m+1) = 3f(m) + 1 = 3f((c1c2…cj)2) + 1 = (10)3.(c1c2…cj)3 + 13 = (c1c2…cj1)3 ⇒ (*)
ñúng cho n l .
V y (*) ñúng cho m i n∈N* và f(n) ñư c xác ñ nh như (*).
Phương pháp 9: phương pháp s d ng ñ o hàm.
Ví d 1: Tìm f: R → R tho mãn: | f(x)- f(y)|2 ≤ | x- y|3 ∀x, y∈R (14).
L i gi i: C ñ nh y, v i x∈R, x ≠ y t (14) ta ñư c:
               2
 f ( x) − f ( y )            f ( x) − f ( y )
                  ≤ x− y ⇒0≤                  ≤      x − y . Vì lim 0 = lim       x − y = 0 nên suy ra
      x− y                        x− y                          x→ y       x→ y


       f ( x) − f ( y )
lim                     = 0 ⇒ f’(y) = 0 ⇒ f(y) = c ∀y∈R (v i c là h ng s ). Th l i th y ñúng.
x→ y        x− y

Ví d 2: Tìm f: R → R có ñ o hàm trên R và tho mãn:
        f(x+y) = f(x) + f(y) + 2xy ∀x, y∈R (15)
L i gi i:
+ Cho x = y = 0 ta ñư c f(0) = 0.


                                                  27
f ( x + y ) − f ( x) f ( y ) + 2 xy f ( y ) − f (0)
+ V i y ≠ 0, c ñ nh x ta ñư c:                          =              =                + 2 x (*). Vì f(x)
                                             y                  y            y −0
có ñ o hàm trên R nên t (*), cho y → 0, suy ra f’(x) = f’(0) + 2x = 2x + c ⇒ f(x) = x2+cx+b
∀x∈R; b, c là các h ng s th c. Th l i th y ñúng.
Phương pháp 10: phương pháp ñ t hàm ph .
       M c ñích chính c a vi c ñ t hàm ph là làm gi m ñ ph c t p c a phương trình
hàm ban ñ u và chuy n ñ i tính ch t hàm s nh m có l i hơn trong gi i toán.
Ví d 1: Tìm f: R → R tho mãn: f(x) ≥ 2007x và f(x+y) ≥ f(x)+f(y) ∀x, y∈R (1).
L i gi i:
D th y f(x) = 2007x là m t hàm s tho mãn (1). ð t g(x) = f(x) - 2007x và thay vào (1) ta
ñư c: g(x) ≥ 0 (a) và g(x+y) ≥ g(x) + g(y) (b) ∀x, y∈R.
+ Cho x = y = 0, t (b) ta ñư c g(0) ≤ 0, k t h p v i (a) suy ra g(0) = 0.
+ Cho x = -y, x∈R, t (a) và (b) ta ñư c g(x) ≥ 0, g(-x) ≥ 0, 0 ≥ g(x) + g(-x); suy ra :
g(x) = g(-x) = 0 ⇒ h(x) = 2007x, ∀x∈R. Th l i th y ñúng.
Ví d 2: Tìm f: R → R liên t c trên R tho mãn:
         f(x+y) = f(x) + f(y) + f(x)f(y) ∀x, y∈ R (2).
L i gi i:
Xét phương trình: λ = 2λ + λ2 có nghi m λ = -1 khác 0.
ð t g(x) = f(x) - (-1) = f(x) + 1. Th vào (18) ta ñư c:
g(x+y) = g(x).g(y) ∀x, y∈R (*).
              t
Cho x = y =     ta ñư c g(t) ≥ 0 ∀t∈R.
              2
Cho x = y = 0 ta ñư c: g(0) = 0 ho c g(0) = 1.
+ N u g(0) = 0 thì (*) suy ra g(x) = 0 ∀x∈R ⇒ f(x) = -1 ∀x∈R. Th l i th y ñúng.
+ N u g(0) = 1: Gi s t n t i a ñ g(a) = 0 thì (*) suy ra g(x) = 0 ∀x∈R. Trái v i gi thi t
g(0) = 1. V y g(x) > 0 ∀x∈R. ð t h(x) = lng(x) ta ñư c :
h(x+y) = h(x) + h(y) (**). T f(x) liên t c trên R suy ra h(x) liên t c trên R. Theo phương
trình hàm Côsi ta ñư c h(x) = cx (v i c là h ng s ) ⇒ f(x) = ecx - 1 ∀x∈R. Khi c = 0 thì
f(x) = -1. V y trong m i trư ng h p f(x) = ecx - 1 ∀x∈R th l i th y ñúng.
Phương pháp 11: S d ng tính liên t c c a hàm s .
      S d ng tính liên t c c a hàm s có 3 con ñư ng chính: Xây d ng bi n t N ñ n
R, ch ng minh hàm s là h ng s , s d ng phương trình hàm Côsi.
Ví d 1 (xây d ng bi n t N ñ n R):
         Tìm hàm f : R → R th a mãn:
         1) f(x) liên t c trên R;
         2) f(1) = 2;

                                                 28
3) f(xy) = f(x)f(y) - f(x+y) +1 ∀x,y∈R.
L i gi i:
Cho x = y = 0 ta ñư c: f(0) = 1.
Cho x = 1, y∈R ta ñư c: f(y+1) = f(y) +1 (a).
T f(0) = 1, f(1) = 2 và (a) quy n p ta suy ra f(n) = n+1 ∀n∈N.
V i n∈N, (a) ⇒ f(-n) = f(-n+1) - 1 = f(-n+2) - 2 =…= f(0) -n = -n + 1.
V y f(z) = z +1 ∀z∈Z.
                           1             1          1
V i ∀n∈N*, 2 = f(1) = f (n. ) = f (n) f ( ) − f (n + ) + 1 (b). M t khác t (a) ta có:
                           n             n          n
      1                  1                  1                 1
f (n + ) = 1 + f (n − 1 + ) = 2 + f (n − 2 + ) = ... = n + f ( ) . Th vào (b) ta ñư c:
      n                  n                  n                 n
   1   1
f ( ) = + 1.
   n   n
                 m                                    m         1             1          1
V i q ∈ ℚ, q =     , m ∈ ℤ, n ∈ ℕ* ta có: f (q ) = f ( ) = f (m. ) = f (m) f ( ) − f (m + ) + 1 =
                 n                                    n         n             n          n
          1             1
= (m + 1)( + 1) − f (m + ) + 1 (c). T (a) ta d dàng ch ng minh ñư c:
          n             n
      1           1
f (m + ) = m + f ( ) . Th vào (c) ta ñư c f(q) = q +1 ∀q∈Q.
      n           n
V i r∈R, t n t i dãy {rn} v i rn∈Q th a mãn lim rn = r . Khi ñó, do f liên t c nên ta có:

f(r) = f(limrn) = limf(rn) = lim(rn+1) = limrn + 1 = r + 1. V y f(x) = x + 1 ∀x∈R. Th l i th y
ñúng.
Ví d 2 (ch ng minh hàm s là h ng s ):
                       1       1
         Tìm hàm f: [0; ] → [0; ] th a mãn:
                       2       2
                                  1
         1) f(x) liên t c trên [0; ]
                                  2
                               1        1
         2) f ( x ) = f ( x 2 + ) ∀x ∈  0;  .
                               4        2
L i gi i:
                          x0 = a
         1               
V i a∈[0; ], xét dãy s :          2  1       .
         2                xn+1 = xn + ∀n ∈ ℕ
                                     4
D ch ng minh {xn} không âm (a).
       1            1 1                     1
x0 ≤     ⇒ x1 ≤ x0 + ≤ . Quy n p suy ra xn ≤ (b).
                 2

       2            4 2                     2


                                                  29
1
xn +1 − xn = ( xn − ) 2 ≥ 0 ⇒ xn +1 ≥ xn ∀n ∈ ℕ (c).
                   2
                             1                                         1
T (a), (b), (c) suy ra xn∈[0; ] và {xn} có gi i h n h u h n là limx n = .
                             2                                         2

                  1                                                   1
V y v i m i a∈[0; ], f(a) = f(x1) = f(x2) =…= limf(xn) = f(limxn) = f( ) = c (c là h ng s ).
                  2                                                   2
Th l i th y ñúng.
Ví d 3 (s d ng phương trình hàm côsi - VMO năm 2006(b ng B)):
Tìm f: R → R liên t c trên R tho mãn: f(x-y).f(y-z).f(z-x)+8 = 0 ∀x, y, z∈R (3).
L i gi i:
                                                                                          −8
Cho x = t, z = -t, y = 0, x∈ R ta ñư c: f(t).f(t).f(-2t) = -8 ⇒ f (−2t ) =                         < 0 ⇒ f(t) < 0
                                                                                       ( f (t )) 2

∀t∈R. ð t g(x) = ln(
                               f ( x)
                                      ) ⇒ f ( x) = −2.e
                                                        g ( x) . Th vào (3) ta ñư c:
                                −2
       g(x-y)+g(y-z)+g(z-x)
-8.e                          = -8 ⇔ g(x-y) + g(y-z) + g(z-x) = 0 (*).
+ Cho x = y = z = 0, t (*) ta ñư c g(0) = 0 (a).
+ Cho y = z = 0, x∈R, t (a) ta ñư c g(x) = g(-x) (b).
T (*) và (b) suy ra g(x-y) + g(y-z) = -g(z-x) = -g(x-z) = g(x-y+y-z) ⇒ g(t+t’) = g(t) + g(t’)
∀t, t’∈R (**). Vì f liên t c trên R nên g(x) cũng liên t c trên R. T (**), theo phương trình
hàm Côsi ta ñư c g(x) = ax ⇒ f(x) = -2.eax = -2.bx (V i b = ea > 0). Th l i th y ñúng.
------------------------------------------------------H t-------------------------------------------------------




                                                        30

More Related Content

What's hot

Anh Xa Lien Tuc Tren Khong Gian Topo
Anh Xa Lien Tuc Tren Khong Gian TopoAnh Xa Lien Tuc Tren Khong Gian Topo
Anh Xa Lien Tuc Tren Khong Gian Topoipaper
 
Do do tich-phan-thai_thuan_quang mearsure and intergral
Do do tich-phan-thai_thuan_quang mearsure and intergralDo do tich-phan-thai_thuan_quang mearsure and intergral
Do do tich-phan-thai_thuan_quang mearsure and intergralBui Loi
 
Topo daicuong1[1]
Topo daicuong1[1]Topo daicuong1[1]
Topo daicuong1[1]Bui Loi
 
Thuật toán berlekamp và đa thức chia đường tròn modulo p
Thuật toán berlekamp và đa thức chia đường tròn modulo pThuật toán berlekamp và đa thức chia đường tròn modulo p
Thuật toán berlekamp và đa thức chia đường tròn modulo pBui Loi
 
Bài Giảng Đại Số Tuyến Tính - ĐH Thăng Long
Bài Giảng Đại Số Tuyến Tính - ĐH Thăng LongBài Giảng Đại Số Tuyến Tính - ĐH Thăng Long
Bài Giảng Đại Số Tuyến Tính - ĐH Thăng LongHoàng Như Mộc Miên
 
Toan a1 -_bai_giang
Toan a1 -_bai_giangToan a1 -_bai_giang
Toan a1 -_bai_giangxuanhoa88
 
Bài toán số học liên quan tới lũy thữa
Bài toán số học liên quan tới lũy thữaBài toán số học liên quan tới lũy thữa
Bài toán số học liên quan tới lũy thữaThế Giới Tinh Hoa
 
Chuyen de toan logic roi rac li thuyet to hop
Chuyen de toan logic  roi rac li thuyet to hopChuyen de toan logic  roi rac li thuyet to hop
Chuyen de toan logic roi rac li thuyet to hoplephucduc06011999
 
đạO hàm và vi phân
đạO hàm và vi phânđạO hàm và vi phân
đạO hàm và vi phânchuateonline
 
bai tap hinh hoc xa anh-pham binh do
bai tap hinh hoc xa anh-pham binh dobai tap hinh hoc xa anh-pham binh do
bai tap hinh hoc xa anh-pham binh doBui Loi
 
Một số vấn đề về không gian Sobolev
Một số vấn đề về không gian SobolevMột số vấn đề về không gian Sobolev
Một số vấn đề về không gian Sobolevnataliej4
 
Bai Tap Hinh Hoc Vi Phan (CoLoiGiai)
Bai Tap Hinh Hoc Vi Phan (CoLoiGiai)Bai Tap Hinh Hoc Vi Phan (CoLoiGiai)
Bai Tap Hinh Hoc Vi Phan (CoLoiGiai)Bui Loi
 
Một số tính chất của vành giao hoán artin
Một số tính chất của vành giao hoán artinMột số tính chất của vành giao hoán artin
Một số tính chất của vành giao hoán artinNOT
 

What's hot (20)

Anh Xa Lien Tuc Tren Khong Gian Topo
Anh Xa Lien Tuc Tren Khong Gian TopoAnh Xa Lien Tuc Tren Khong Gian Topo
Anh Xa Lien Tuc Tren Khong Gian Topo
 
Do do tich-phan-thai_thuan_quang mearsure and intergral
Do do tich-phan-thai_thuan_quang mearsure and intergralDo do tich-phan-thai_thuan_quang mearsure and intergral
Do do tich-phan-thai_thuan_quang mearsure and intergral
 
Topo daicuong1[1]
Topo daicuong1[1]Topo daicuong1[1]
Topo daicuong1[1]
 
Bài tập hàm biến phức
Bài tập hàm biến phứcBài tập hàm biến phức
Bài tập hàm biến phức
 
Thuật toán berlekamp và đa thức chia đường tròn modulo p
Thuật toán berlekamp và đa thức chia đường tròn modulo pThuật toán berlekamp và đa thức chia đường tròn modulo p
Thuật toán berlekamp và đa thức chia đường tròn modulo p
 
Luận văn: Lý thuyết đồ thị với các bài toán phổ thông, HAY, 9đ
Luận văn: Lý thuyết đồ thị với các bài toán phổ thông, HAY, 9đLuận văn: Lý thuyết đồ thị với các bài toán phổ thông, HAY, 9đ
Luận văn: Lý thuyết đồ thị với các bài toán phổ thông, HAY, 9đ
 
Bài Giảng Đại Số Tuyến Tính - ĐH Thăng Long
Bài Giảng Đại Số Tuyến Tính - ĐH Thăng LongBài Giảng Đại Số Tuyến Tính - ĐH Thăng Long
Bài Giảng Đại Số Tuyến Tính - ĐH Thăng Long
 
Toan a1 -_bai_giang
Toan a1 -_bai_giangToan a1 -_bai_giang
Toan a1 -_bai_giang
 
Luận án: Mở rộng của lớp Môđun giả nội xạ và vành liên quan, HAY
Luận án: Mở rộng của lớp Môđun giả nội xạ và vành liên quan, HAYLuận án: Mở rộng của lớp Môđun giả nội xạ và vành liên quan, HAY
Luận án: Mở rộng của lớp Môđun giả nội xạ và vành liên quan, HAY
 
Bdt thuần nhất
Bdt thuần nhấtBdt thuần nhất
Bdt thuần nhất
 
Bài toán số học liên quan tới lũy thữa
Bài toán số học liên quan tới lũy thữaBài toán số học liên quan tới lũy thữa
Bài toán số học liên quan tới lũy thữa
 
Luận văn: Tính chất định tính của hàm đa trị và ứng dụng, HOT
Luận văn: Tính chất định tính của hàm đa trị và ứng dụng, HOTLuận văn: Tính chất định tính của hàm đa trị và ứng dụng, HOT
Luận văn: Tính chất định tính của hàm đa trị và ứng dụng, HOT
 
Chuyen de toan logic roi rac li thuyet to hop
Chuyen de toan logic  roi rac li thuyet to hopChuyen de toan logic  roi rac li thuyet to hop
Chuyen de toan logic roi rac li thuyet to hop
 
Bt dai so hoang
Bt dai so hoangBt dai so hoang
Bt dai so hoang
 
đạO hàm và vi phân
đạO hàm và vi phânđạO hàm và vi phân
đạO hàm và vi phân
 
bai tap hinh hoc xa anh-pham binh do
bai tap hinh hoc xa anh-pham binh dobai tap hinh hoc xa anh-pham binh do
bai tap hinh hoc xa anh-pham binh do
 
Dãy số và giới hạn
Dãy số và giới hạnDãy số và giới hạn
Dãy số và giới hạn
 
Một số vấn đề về không gian Sobolev
Một số vấn đề về không gian SobolevMột số vấn đề về không gian Sobolev
Một số vấn đề về không gian Sobolev
 
Bai Tap Hinh Hoc Vi Phan (CoLoiGiai)
Bai Tap Hinh Hoc Vi Phan (CoLoiGiai)Bai Tap Hinh Hoc Vi Phan (CoLoiGiai)
Bai Tap Hinh Hoc Vi Phan (CoLoiGiai)
 
Một số tính chất của vành giao hoán artin
Một số tính chất của vành giao hoán artinMột số tính chất của vành giao hoán artin
Một số tính chất của vành giao hoán artin
 

Similar to Bộ sách về phương trình hàm

Tổng hợp bồi dưỡng học sinh giỏi
Tổng hợp bồi dưỡng học sinh giỏiTổng hợp bồi dưỡng học sinh giỏi
Tổng hợp bồi dưỡng học sinh giỏiNhập Vân Long
 
11 phuong phap giai pth
11 phuong phap giai pth11 phuong phap giai pth
11 phuong phap giai pthPhuc Nguyen
 
Cac phuong phap giai pt ham thuong dung
Cac phuong phap giai pt ham thuong dungCac phuong phap giai pt ham thuong dung
Cac phuong phap giai pt ham thuong dungljmonking
 
chuyen de tich phan on thi dai hoc
chuyen de tich phan on thi dai hocchuyen de tich phan on thi dai hoc
chuyen de tich phan on thi dai hocHoàng Thái Việt
 
toan boi duong HSG ntquang.net
toan boi duong HSG ntquang.nettoan boi duong HSG ntquang.net
toan boi duong HSG ntquang.netDuy Duy
 
Chuyen de-ham-so-bac-i-va-ii
Chuyen de-ham-so-bac-i-va-iiChuyen de-ham-so-bac-i-va-ii
Chuyen de-ham-so-bac-i-va-iiNguyen Van Tai
 
Chuyên đề 2 hàm số bậc i và ii
Chuyên đề 2 hàm số bậc i và iiChuyên đề 2 hàm số bậc i và ii
Chuyên đề 2 hàm số bậc i và iiphamchidac
 
phương trình hàm.pdf
phương trình hàm.pdfphương trình hàm.pdf
phương trình hàm.pdfNguyenTanBinh4
 
Ôn thi THPT Quốc Gia môn Toán về nguyên hàm và tích phân
Ôn thi THPT Quốc Gia môn Toán về nguyên hàm và tích phânÔn thi THPT Quốc Gia môn Toán về nguyên hàm và tích phân
Ôn thi THPT Quốc Gia môn Toán về nguyên hàm và tích phânLinh Nguyễn
 
Bt gioi han_ham_so_6893
Bt gioi han_ham_so_6893Bt gioi han_ham_so_6893
Bt gioi han_ham_so_6893irisgk10
 
Chuyen de boi_duong_hoc_sinh_gioi_lop_12_2802
Chuyen de boi_duong_hoc_sinh_gioi_lop_12_2802Chuyen de boi_duong_hoc_sinh_gioi_lop_12_2802
Chuyen de boi_duong_hoc_sinh_gioi_lop_12_2802baolanchi
 
De thi hoc ki 2 k12 nam 0910
De thi hoc ki 2 k12 nam 0910De thi hoc ki 2 k12 nam 0910
De thi hoc ki 2 k12 nam 0910lvquy
 
De Thi Hoc Ki 2 K12 Nam 0910
De Thi Hoc Ki 2 K12 Nam 0910De Thi Hoc Ki 2 K12 Nam 0910
De Thi Hoc Ki 2 K12 Nam 0910lvquy
 
De thi hoc ki 2 k12 nam 0910
De thi hoc ki 2 k12 nam 0910De thi hoc ki 2 k12 nam 0910
De thi hoc ki 2 k12 nam 0910lvquy
 
De thi hoc ki 2 k12 nam 0910
De thi hoc ki 2 k12 nam 0910De thi hoc ki 2 k12 nam 0910
De thi hoc ki 2 k12 nam 0910lvquy
 
De thi hoc ki 2 k12 nam 0910
De thi hoc ki 2 k12 nam 0910De thi hoc ki 2 k12 nam 0910
De thi hoc ki 2 k12 nam 0910lvquy
 
Tổng hợp công thức giải nhanh trắc nghiệm toán THPT Quốc gia 2018
Tổng hợp công thức giải nhanh trắc nghiệm toán THPT Quốc gia 2018Tổng hợp công thức giải nhanh trắc nghiệm toán THPT Quốc gia 2018
Tổng hợp công thức giải nhanh trắc nghiệm toán THPT Quốc gia 2018Maloda
 
Giaipt nghiemnguyen
Giaipt nghiemnguyenGiaipt nghiemnguyen
Giaipt nghiemnguyenhonghoi
 

Similar to Bộ sách về phương trình hàm (20)

Tổng hợp bồi dưỡng học sinh giỏi
Tổng hợp bồi dưỡng học sinh giỏiTổng hợp bồi dưỡng học sinh giỏi
Tổng hợp bồi dưỡng học sinh giỏi
 
11 phuong phap giai pth
11 phuong phap giai pth11 phuong phap giai pth
11 phuong phap giai pth
 
Cac phuong phap giai pt ham thuong dung
Cac phuong phap giai pt ham thuong dungCac phuong phap giai pt ham thuong dung
Cac phuong phap giai pt ham thuong dung
 
Chuyen de hsg
Chuyen de hsgChuyen de hsg
Chuyen de hsg
 
chuyen de tich phan on thi dai hoc
chuyen de tich phan on thi dai hocchuyen de tich phan on thi dai hoc
chuyen de tich phan on thi dai hoc
 
toan boi duong HSG ntquang.net
toan boi duong HSG ntquang.nettoan boi duong HSG ntquang.net
toan boi duong HSG ntquang.net
 
Chuyen de-ham-so-bac-i-va-ii
Chuyen de-ham-so-bac-i-va-iiChuyen de-ham-so-bac-i-va-ii
Chuyen de-ham-so-bac-i-va-ii
 
Chuyên đề 2 hàm số bậc i và ii
Chuyên đề 2 hàm số bậc i và iiChuyên đề 2 hàm số bậc i và ii
Chuyên đề 2 hàm số bậc i và ii
 
phương trình hàm.pdf
phương trình hàm.pdfphương trình hàm.pdf
phương trình hàm.pdf
 
Ôn thi THPT Quốc Gia môn Toán về nguyên hàm và tích phân
Ôn thi THPT Quốc Gia môn Toán về nguyên hàm và tích phânÔn thi THPT Quốc Gia môn Toán về nguyên hàm và tích phân
Ôn thi THPT Quốc Gia môn Toán về nguyên hàm và tích phân
 
Bt gioi han_ham_so_6893
Bt gioi han_ham_so_6893Bt gioi han_ham_so_6893
Bt gioi han_ham_so_6893
 
Chuyen de boi_duong_hoc_sinh_gioi_lop_12_2802
Chuyen de boi_duong_hoc_sinh_gioi_lop_12_2802Chuyen de boi_duong_hoc_sinh_gioi_lop_12_2802
Chuyen de boi_duong_hoc_sinh_gioi_lop_12_2802
 
De thi hoc ki 2 k12 nam 0910
De thi hoc ki 2 k12 nam 0910De thi hoc ki 2 k12 nam 0910
De thi hoc ki 2 k12 nam 0910
 
De Thi Hoc Ki 2 K12 Nam 0910
De Thi Hoc Ki 2 K12 Nam 0910De Thi Hoc Ki 2 K12 Nam 0910
De Thi Hoc Ki 2 K12 Nam 0910
 
De thi hoc ki 2 k12 nam 0910
De thi hoc ki 2 k12 nam 0910De thi hoc ki 2 k12 nam 0910
De thi hoc ki 2 k12 nam 0910
 
De thi hoc ki 2 k12 nam 0910
De thi hoc ki 2 k12 nam 0910De thi hoc ki 2 k12 nam 0910
De thi hoc ki 2 k12 nam 0910
 
De thi hoc ki 2 k12 nam 0910
De thi hoc ki 2 k12 nam 0910De thi hoc ki 2 k12 nam 0910
De thi hoc ki 2 k12 nam 0910
 
Nguyen ham
Nguyen hamNguyen ham
Nguyen ham
 
Tổng hợp công thức giải nhanh trắc nghiệm toán THPT Quốc gia 2018
Tổng hợp công thức giải nhanh trắc nghiệm toán THPT Quốc gia 2018Tổng hợp công thức giải nhanh trắc nghiệm toán THPT Quốc gia 2018
Tổng hợp công thức giải nhanh trắc nghiệm toán THPT Quốc gia 2018
 
Giaipt nghiemnguyen
Giaipt nghiemnguyenGiaipt nghiemnguyen
Giaipt nghiemnguyen
 

More from Thế Giới Tinh Hoa

Cách chụp ảnh công ty đẹp 2019
Cách chụp ảnh công ty đẹp 2019Cách chụp ảnh công ty đẹp 2019
Cách chụp ảnh công ty đẹp 2019Thế Giới Tinh Hoa
 
Bảng báo giá sản phẩm rèm bạch dương
Bảng báo giá sản phẩm rèm bạch dươngBảng báo giá sản phẩm rèm bạch dương
Bảng báo giá sản phẩm rèm bạch dươngThế Giới Tinh Hoa
 
Album sổ mẫu Rèm cửa Bạch Dương
Album sổ mẫu Rèm cửa Bạch DươngAlbum sổ mẫu Rèm cửa Bạch Dương
Album sổ mẫu Rèm cửa Bạch DươngThế Giới Tinh Hoa
 
Cách tắm cho bé vào mùa đông
Cách tắm cho bé vào mùa đôngCách tắm cho bé vào mùa đông
Cách tắm cho bé vào mùa đôngThế Giới Tinh Hoa
 
Giáo trình tự học illustrator cs6
Giáo trình tự học illustrator cs6  Giáo trình tự học illustrator cs6
Giáo trình tự học illustrator cs6 Thế Giới Tinh Hoa
 
Nữ quái sân trườngtruonghocso.com
Nữ quái sân trườngtruonghocso.comNữ quái sân trườngtruonghocso.com
Nữ quái sân trườngtruonghocso.comThế Giới Tinh Hoa
 
Những chàng trai xấu tính nguyễn nhật ánhtruonghocso.com
Những chàng trai xấu tính  nguyễn nhật ánhtruonghocso.comNhững chàng trai xấu tính  nguyễn nhật ánhtruonghocso.com
Những chàng trai xấu tính nguyễn nhật ánhtruonghocso.comThế Giới Tinh Hoa
 
Những bài văn mẫu dành cho học sinh lớp 10truonghocso.com
Những bài văn mẫu dành cho học sinh lớp 10truonghocso.comNhững bài văn mẫu dành cho học sinh lớp 10truonghocso.com
Những bài văn mẫu dành cho học sinh lớp 10truonghocso.comThế Giới Tinh Hoa
 

More from Thế Giới Tinh Hoa (20)

Cách chụp ảnh công ty đẹp 2019
Cách chụp ảnh công ty đẹp 2019Cách chụp ảnh công ty đẹp 2019
Cách chụp ảnh công ty đẹp 2019
 
Lỗi web bachawater
Lỗi web bachawaterLỗi web bachawater
Lỗi web bachawater
 
Bảng báo giá sản phẩm rèm bạch dương
Bảng báo giá sản phẩm rèm bạch dươngBảng báo giá sản phẩm rèm bạch dương
Bảng báo giá sản phẩm rèm bạch dương
 
Album sổ mẫu Rèm cửa Bạch Dương
Album sổ mẫu Rèm cửa Bạch DươngAlbum sổ mẫu Rèm cửa Bạch Dương
Album sổ mẫu Rèm cửa Bạch Dương
 
thong tin lam viec tren lamchame
thong tin lam viec tren lamchamethong tin lam viec tren lamchame
thong tin lam viec tren lamchame
 
Cách tắm cho bé vào mùa đông
Cách tắm cho bé vào mùa đôngCách tắm cho bé vào mùa đông
Cách tắm cho bé vào mùa đông
 
Giáo trình tự học illustrator cs6
Giáo trình tự học illustrator cs6  Giáo trình tự học illustrator cs6
Giáo trình tự học illustrator cs6
 
Nang luc truyen thong
Nang luc truyen thongNang luc truyen thong
Nang luc truyen thong
 
Huongdansudung izishop
Huongdansudung izishopHuongdansudung izishop
Huongdansudung izishop
 
Ho so nang luc cong ty
Ho so nang luc cong tyHo so nang luc cong ty
Ho so nang luc cong ty
 
seo contract
seo contractseo contract
seo contract
 
di google cong
di google congdi google cong
di google cong
 
E1 f4 bộ binh
E1 f4 bộ binhE1 f4 bộ binh
E1 f4 bộ binh
 
E2 f2 bộ binh
E2 f2 bộ binhE2 f2 bộ binh
E2 f2 bộ binh
 
E3 f1 bộ binh
E3 f1 bộ binhE3 f1 bộ binh
E3 f1 bộ binh
 
E2 f1 bộ binh
E2 f1 bộ binhE2 f1 bộ binh
E2 f1 bộ binh
 
E1 f1 bộ binh
E1 f1 bộ binhE1 f1 bộ binh
E1 f1 bộ binh
 
Nữ quái sân trườngtruonghocso.com
Nữ quái sân trườngtruonghocso.comNữ quái sân trườngtruonghocso.com
Nữ quái sân trườngtruonghocso.com
 
Những chàng trai xấu tính nguyễn nhật ánhtruonghocso.com
Những chàng trai xấu tính  nguyễn nhật ánhtruonghocso.comNhững chàng trai xấu tính  nguyễn nhật ánhtruonghocso.com
Những chàng trai xấu tính nguyễn nhật ánhtruonghocso.com
 
Những bài văn mẫu dành cho học sinh lớp 10truonghocso.com
Những bài văn mẫu dành cho học sinh lớp 10truonghocso.comNhững bài văn mẫu dành cho học sinh lớp 10truonghocso.com
Những bài văn mẫu dành cho học sinh lớp 10truonghocso.com
 

Bộ sách về phương trình hàm

  • 1. CÁC PHƯƠNG PHÁP GI I PHƯƠNG TRÌNH HÀM THƯ NG DÙNG Phương pháp 1: H s b t ñ nh. Nguyên t c chung: +) D a vào ñi u ki n bài toán, xác ñ nh ñư c d ng c a f(x), thư ng là f(x) = ax + b ho c f(x) = ax2+ bx + c. +) ð ng nh t h s ñ tìm f(x). +) Ch ng minh r ng m i h s khác c a f(x) ñ u không th a mãn ñi u ki n bài toán. Ví d 1: Tìm f : R → R th a mãn: f ( x f ( y ) + x ) = xy + f ( x ) ∀x, y ∈ R (1) . L i gi i: x = 1 Thay  vào (18) ta ñư c: f ( f ( y ) + 1) = y + f (1) ( a ) . y∈R ( ) Thay y = − f (1) − 1 vào (a) suy ra: f f ( − f (1) − 1) + 1 = −1 . ð t a = f ( − f (1) − 1) + 1 ta ñư c: f ( a ) = −1 . y = a Ch n  ta ñư c: f ( x f ( a ) + x ) = xa + f ( x ) ⇒ xa + f ( x ) = f ( 0 ) . x ∈ R ð t f ( 0 ) = b ⇒ f ( x ) = −a x + b . Th vào (1) và ñ ng nh t h s ta ñư c: a = 1 a 2 = 1   f ( x) = x  ⇒   a = −1 ⇒   . − a b − a = −a   f ( x) = −x  b = 0 V y có hai hàm s c n tìm là f ( x ) = x và f ( x ) = − x . Ví d 2: Tìm f : R → R th a mãn: f ( f ( x ) + y ) = y f ( x − f ( y ) ) ∀x, y ∈ R ( 2 ) . L i gi i: Cho y = 0; x ∈ R : (2) ⇒ f ( f ( x ) ) = 0 ∀x ∈ R ( a ) . ( ) Cho x = f ( y ) : (2) ⇒ f f ( f ( y ) ) + y = y f ( 0 ) ( a ' ) . ( a ) + ( a' ) ⇒ f ( y ) = y f ( 0) . ð t f ( 0 ) = a ⇒ f ( y ) = ay ∀y ∈ R . Th l i (2) ta ñư c: a 2 ( x 2 + y 2 ) + a ( y − x y ) = 0 ∀x, y ∈ R ⇔ a = 0 ⇒ f ( x ) = 0 ∀x ∈ R . V y có duy nh t hàm s f ( x ) = 0 th a mãn bài toán. Ví d 3: Tìm f , g : R → R th a mãn: 2 f ( x ) − g ( x ) = f ( y ) − y ∀x, y ∈ R  (a)  .  f ( x) g ( x) ≥ x +1  ∀x ∈ R (b ) L i gi i: Cho x = y ∈ R khi ñó ( a ) ⇒ f ( x ) = g ( x ) − x .Thay l i (a) ta ñư c: 1
  • 2. g ( x ) = 2 x − 2 y + g ( y ) ∀x, y ∈ R (c). Cho y = 0; x ∈ R : t (c) ta ñư c: g ( x ) = 2 x + g ( 0 ) . ð t g ( 0 ) = a ta ñư c: g ( x ) = 2 x + a , f ( x ) = x + a . Th vào (a), (b) ta ñư c: 2 x + a = 2 x + a  (a), (b) ⇔  ( ∀x ∈ R ) ⇔ 2 x 2 + ( 3a − 1) x + a 2 − 1 ≥ 0 ∀x ∈ R (  x + a )( 2 x + a ) ≥ x + 1 2 ⇔ ( a − 3) ≤ 0 ⇔ a = 3 . V y f ( x ) = x + 3 ; g ( x ) = 2 x + 3 . Ví d 4: ða th c f(x) xác ñ nh v i ∀x ∈ ℝ và th a mãn ñi u ki n: 2 f ( x ) + f (1 − x ) = x 2 , ∀x ∈ ℝ (1). Tìm f(x). L i gi i: Ta nh n th y v trái c a bi u th c dư i d u f là b c nh t: x, 1 – x v ph i là b c hai x2. V y f(x) ph i có d ng: f(x) = ax2 + bx + c. Khi ñó (1) tr thành: 2(ax2 + bx + c) + a(1 – x)2 + b(1 – x) + c = x2 ∀x ∈ ℝ do ñó: 3ax2 + (b – 2a)x + a + b + 3c = x2, ∀x ∈ ℝ  1 a = 3 3a = 1    2 ð ng nh t các h s , ta thu ñư c: b − 2a = 0 ⇔ b = a + b + 3c = 0  3   1 c = − 3  1 V y: f ( x) = ( x 2 + 2 x − 1) 3 Th l i ta th y hi n nhiên f(x) th a mãn ñi u ki n bài toán. Ta ph i ch ng minh m i hàm s khác f(x) s không th a mãn ñi u ki n bài toán: Th t v y gi s còn hàm s g(x) khác f(x) th a mãn ñi u ki n bài toán. Do f(x) không trùng v i g(x) nên ∃x0 ∈ ℝ : g ( x0 ) ≠ f ( x0 ) . Do g(x) th a mãn ñi u ki n bài toán nên: 2 g ( x) + g (1 − x) = x 2 , ∀x ∈ ℝ Thay x b i x0 ta ñư c: 2 g ( x0 ) + g (1 − x0 ) = x0 2 Thay x b i 1 –x0 ta ñư c: 2 g (1 − x0 ) + g ( x0 ) = (1 − x0 ) 2 1 T hai h th c này ta ñư c: g ( x0 ) = ( x0 2 + 2 x0 − 1) = f ( x0 ) 3 ði u này mâu thu n v i g ( x0 ) ≠ f ( x0 ) 1 V y phương trình có nghi m duy nh t là f ( x) = ( x 2 + 2 x − 1) 3 2
  • 3. Nh n xét: N u ta ch d ñoán f(x) có d ng nào ñó thì ph i ch ng minh s duy nh t c a các hàm s tìm ñư c. Ví d 5: Hàm s y = f(x) xác ñ nh, liên t c v i ∀x ∈ ℝ và th a mãn ñi u ki n: f(f(x)) = f(x) + x, ∀x ∈ ℝ Hãy tìm hai hàm s như th . L i gi i: Ta vi t phương trình ñã cho dư i d ng f(f(x)) – f(x) = x (1). V ph i c a phương trình là m t hàm s tuy n tính vì v y ta nên gi s r ng hàm s c n tìm có d ng: f(x) = ax + b. Khi ñó (1) tr thành: a( ax + b) + b – (ax + b) = x , ∀x ∈ ℝ hay (a2 –a )x + ab = x, ∀x ∈ ℝ   a 2 − a = 1 a = 1 + 5 a = 1 − 5 1± 5 ñ ng nh t h s ta ñư c:  ⇔ 2 ∨ 2 ⇒ f ( x) = x. ab = 0 b = 0 b = 0 2   Hi n nhiên hai hàm s trên th a mãn ñi u ki n bài toán (vi c ch ng minh s duy nh t dành cho ngư i ñ c). Ví d 6: Hàm s f : ℤ → ℤ th a mãn ñ ng th i các ñi u ki n sau: a ) f ( f ( n)) = n, ∀n ∈ ℤ (1) b) f ( f (n + 2) + 2) = n, ∀n ∈ ℤ (2) c) f (0) = 1 (3) Tìm giá tr f(1995), f(-2007). L i gi i: Cũng nh n xét và lý lu n như các ví d trư c, ta ñưa ñ n f(n) ph i có d ng: f(n) = an +b. Khi ñó ñi u ki n (1) tr thành: a 2 n + ab + b = n, ∀n ∈ ℤ a 2 = 1 a = 1 a = −1 ð ng nh t các h s , ta ñư c:  ⇔ ∨ ab + b = 0 b = 0 b = 0 a = 1 V i  ta ñư c f(n) = n. Trư ng h p này lo i vì không th a mãn (2). b = 0 a = −1 V i  ta ñư c f(n) = -n + b. T ñi u ki n (3) cho n = 0 ta ñư c b = 1. b = 0 V y f(n) = -n + 1. Hi n nhiên hàm s này th a mãn ñi u ki n bài toán. Ta ph i ch ng minh f(n) = -n +1 là hàm duy nh t th a mãn ñi u ki n bài toán: Th t v y gi s t n t i hàm g(n) khác f(n) cũng th a mãn ñi u ki n bài toán. T (3) suy ra f(0) = g(0) = 1, f(1) = g(1) = 0. S d ng ñi u ki n (1) và (2) ta nh n ñư c: g(g(n)) = g(g(n+2)+2) ∀n ∈ℤ . 3
  • 4. do ñó g(g(g(n))) = g(g(g(n+2)+2)) ∀n ∈ℤ Hay g(n) = g(n+2)+2 ∀n ∈ℤ . Gi s n0 là s t nhiên bé nh t làm cho f (n0 ) ≠ g (n0 ) Do f(n) cũng th a mãn (4) nên ta có: g (n0 − 2) = g (n0 ) + 2 = f (n0 ) + 2 = f (n0 − 2) ⇔ g (n0 − 2) = f (n0 − 2) Mâu thu n v i ñi u ki n n0 là s t nhiên bé nh t th a mãn (5). V y f(n) = g(n), ∀n ∈ ℕ Ch ng minh tương t ta cũng ñư c f(n) = g(n) v i m i n nguyên âm. V y f(n) = 1 – n là nghi m duy nh t. T ñó tính ñư c f(1995), f(-2007). BÀI T P Bài 1: Tìm t t c các hàm s f : ℝ → ℝ th a mãn ñi u ki n: f ( x + y ) + f ( x − y ) − 2 f ( x) f (1 + y ) = 2 xy (3 y − x 2 ), ∀x, y ∈ ℝ . ðáp s : f(x) = x3. Bài 2: Hàm s f : ℕ → ℕ th a mãn ñi u ki n f(f(n)) + f(n) = 2n + 3, ∀n ∈ ℕ. Tìm f(2005). ðáp s : 2006. Bài 3: Tìm t t c các hàm f : ℕ → ℕ sao cho: f ( f (n)) + ( f (n))2 = n 2 + 3n + 3, ∀n ∈ ℕ. ðáp s : f(n) = n + 1.  x −1   1− x  8  2  Bài 4: Tìm các hàm f : ℝ → ℝ n u: 3 f  −5f  = , ∀x ∉ 0, − ,1, 2   3x + 2   x − 2  x −1  3  28 x + 4 ðáp s : f ( x) = 5x Bài 5: Tìm t t c các ña th c P(x) ∈ ℝ [ x] sao cho: P(x + y) = P(x) + P(y) + 3xy(x + y), ∀x , y ∈ ℝ ðáp s : P(x) = x3 + cx. Phương pháp 2: phương pháp th . 2.1. Th n t o PTH m i:  2x +1  Ví d 1: Tìm f: R{2} → R th a mãn: f   = x + 2 x ∀x ≠ 1 (1) . 2  x −1   2x +1  L i gi i: ð t t =   ⇒ MGT t = R {2} (t p xác ñ nh c a f). Ta ñư c:  x −1  x ≠1 t +1 3t 2 − 3 x= th vào (1): f (t ) = 2 ∀t ≠ 2 . Th l i th y ñúng. t−2 (t − 2) 4
  • 5. 3x 2 − 3 V y hàm s c n tìm có d ng f ( x) = 2 . ( x − 2) Nh n xét: + Khi ñ t t, c n ki m tra gi thi t MGT t ⊃ D . V i gi thi t ñó m i ñ m b o tính ch t: “Khi x∈Dx t ch y kh p các giá tr c a t thì x = t cũng ch y kh p t p xác ñ nh c a f”.  3x 2 − 3  2 ( x ≠ 2) + Trong ví d 1, n u f: R → R thì có vô s hàm f d ng: f ( x) =  ( x − 2 ) (v i a∈R  a ( x = 2) tùy ý). Ví d 2: Tìm hàm f : ( −∞; −1] ∪ ( 0;1] → R th a mãn: f ( x − x 2 − 1) = x + x 2 − 1 ∀ x ≥ 1 ( 2 ) . x − t ≥ 0  L i gi i: ð t t = x − x 2 − 1 ⇔ x 2 − 1 = x − t ⇔  2 2 x −1 = ( x − t )  x ≥ t x ≥ t  t2 +1 t ≤ −1 ⇔ 2 2 2 ⇔ 2 t +1 . H có nghi m x ⇔ ≥t ⇔   x − 1 = x − 2 xt + t x = 2t 0 < t ≤ 1  2t ⇒ t ∈ ( −∞; −1] ∪ ( 0;1] . V y MGT t = D = ( −∞; −1] ∪ ( 0;1] . x ≥1 1 1 V i t = x − x 2 − 1 thì x + x 2 − 1 = ⇒ f (t ) = th a mãn (2). t t 1 V y f ( x) = là hàm s c n tìm. x 2   3x − 1  x + 1 Ví d 3: Tìm f : R  ;3 → R th a mãn: f  = ∀x ≠ 1, x ≠ −2 ( 3) . 3   x + 2  x −1 3x − 1 2  2t + 1 t+4 L i gi i: ð t t = ⇒ MGT t = R  ;3 ⇒ x = th vào (4) ta ñư c: f (t ) = x+2 ( x ≠2) x ≠1 3  3−t 3t − 2 x+4 th a mãn (3). V y hàm s c n tìm là: f ( x) = . 3x − 2 Ví d 4: Tìm f : ( 0; + ∞ ) → ( 0; + ∞ ) th a mãn: x f ( x f ( y )) = f ( f ( y )) ∀x, y ∈ ( 0; + ∞ ) (4) . L i gi i: Cho y = 1, x ∈ ( 0; + ∞ ) ta ñư c: x f ( x f (1)) = f ( f (1)) . 1 1 Cho x = ta ñư c: f ( f (1) = 1⇒ x f ( x f (1)) = 1 ⇒ f ( x f (1)) = . ð t: f (1) x 5
  • 6. f (1) a t = x. f (1) ⇒ f (t ) = ⇒ f (t ) = (v i a = f (1) ). Vì f (1) ∈ ( 0; + ∞ ) ⇒ MGT t = ( 0; + ∞ ) . t t x∈( 0; +∞ ) a a V y f ( x) = . Th l i th y ñúng ( a > 0 ) . Hàm s c n tìm là: f ( x) = v i ( a > 0 ) . x x Ví d 5: Tìm hàm f: ( 0; + ∞ ) → ( 0; + ∞ ) th a mãn: 1 3 3 f (1) = ; f ( xy ) = f ( x). f   + f ( y ). f   ∀x, y ∈ ( 0; + ∞ ) ( 5 ) . 2  y x L i gi i: 1 Cho x = 1; y = 3 ta ñư c: f ( 3) = . 2 3 Cho x = 1; y ∈ ( 0; + ∞ ) ta ñư c: f ( y ) = f   . Th l i (5) ta ñư c:  y 3 f ( xy ) = 2 f ( x) f ( y ) ∀x, y ∈ ( 0; + ∞ ) (5') . Thay y b i ta ñư c: x 2 3 1 2 f ( 3) = 2 f ( x )) f   ⇒   = ( f ( x ) ) . Th l i th y ñúng. x 2 1 V y hàm s c n tìm là: f ( x ) = ∀x > 0 . 2 Ví d 6: Tìm hàm f: R → R th a mãn: ( x − y ) f ( x + y ) − ( x + y ) f ( x − y ) = 4 xy ( x 2 + y 2 ) ∀x, y ∈ R ( 6) . L i gi i: Ta có: ( 6) ⇔ ( x − y ) f ( x + y ) − ( x + y ) f ( x − y ) = 1 2 1 2 = ( x + y ) − ( x − y )  + ( x + y ) + ( x − y )   ( x + y ) + ( x − y )  −  ( x + y ) − ( x − y )       4  4    u = x − y 1 ð t  v = x + y 2 ( ta ñư c: v f ( u ) − u f ( v ) = ( u + v )( u − v ) ( u + v ) − ( u − v ) 4 2 ) ⇒ v f ( u ) − u f ( v ) = u 3v − v 3u ⇔ v ( f ( u ) − u 3 ) = u ( f ( v ) − v3 ) + V i uv ≠ 0 ta có: f ( u ) − u 3 f ( v ) − v3 f (u ) − u3 = ∀u , v ∈ R* ⇒ = a ⇒ f ( u ) = au + u 3 ∀u ≠ 0 . u v u + V i u = 0; v ≠ 0 suy ra: f ( u ) − u 3 = 0 ⇔ f ( u ) = u 3 ⇒ f ( 0 ) = 0 . Hàm f ( u ) = au + u 3 th a mãn f ( 0 ) = 0 . V y f ( u ) = au + u 3 ∀u ∈ R Hàm s c n tìm là: f ( x ) = ax + x3 ( a ∈ R ) . Th l i th y ñúng. 2.2. Th n t o ra h PTH m i: 6
  • 7. Ví d 1: Tìm hàm f: R → R th a mãn: f ( x ) + x f ( − x ) = x + 1 ∀x ∈ R (1) . L i gi i: ð t t = − x ta ñư c: f ( −t ) − t f ( t ) = −t + 1 ∀t ∈ R (1) . Ta có h :  f ( x) + x f (−x) = x +1   ⇒ f ( x ) = 1 . Th l i hàm s c n tìm là: f ( x ) = 1 . − x f ( x ) + f ( − x ) = − x + 1   x −1  Ví d 2: Tìm hàm s f : R { 0,1 } → R Th a mãn: f ( x ) + f   = 1 + x ∀x ∈ R * ( 2) .  x  x −1 L i gi i: ð t x1 = , ( 2 ) ⇔ f ( x ) + f ( x1 ) = 1 + x . x x1 − 1 1 ð t x2 = = , ( 2 ) ⇔ f ( x1 ) + f ( x2 ) = 1 + x1 . x1 x −1 x2 − 1 ð t x3 = = x, ( 2 ) ⇔ f ( x2 ) + f ( x ) = 1 + x2 . x2  f ( x1 ) + f ( x ) = 1 + x  1 + x − x1 + x2 1  1 1  Ta có h  f ( x2 ) + f ( x1 ) = 1 + x1 ⇒ f ( x ) = = x+ +  . Th l i th y  2 2 x 1− x   f ( x ) + f ( x2 ) = 1 + x2 1 1 1  ñúng. V y hàm s c n tìm có d ng: f ( x ) =  x + + . 2 x 1− x   x −1  Ví d 3: Tìm hàm s f : R { − 1;0;1 } → R th a mãn: x f ( x ) + 2 f   = 1 ∀x ≠ −1 ( 3) .  x +1  L i gi i: x −1 ð t x1 = , ( 3) ⇒ x f ( x ) + 2 f ( x1 ) = 1 . x +1 x1 − 1 1 ð t x2 = = − , ( 3) ⇒ x 1 f ( x1 ) + 2 f ( x2 ) = 1 . x1 + 1 x x2 − 1 x + 1 ð t x3 = = , ( 3) ⇒ x2 f ( x2 ) + 2 f ( x3 ) = 1 . x2 + 1 x − 1 x3 − 1 ð t x4 = = x , ( 3) ⇒ x3 f ( x3 ) + 2 f ( x ) = 1 . x3 + 1  x f ( x ) + 2 f ( x1 ) = 1   x1 f ( x1 ) + 2 f ( x2 ) = 1 4 x2 − x + 1 Ta có h  ⇒ f ( x) = . Th l i th y ñúng.  x2 f ( x2 ) + 2 f ( x3 ) = 1 5 x ( x − 1) x f x + 2 f x = 1  3 ( 3) ( ) 7
  • 8. 4 x2 − x + 1 V y hàm s c n tìm là: f ( x ) = . 5 x ( x − 1) BÀI T P  1 1) Tìm f : R { 1 } → R th a mãn: f  1 +  = x 2 + 1 ∀x ∈ R .  x  a  b − ax  x2 a 2) Tìm f : R  −  → R th a mãn: f  = 4 ∀x ≠ − (a, b là h ng s cho  b  bx + a  x + 1 b trư c và ab ≠ 0 ). 3) Tìm f : R → R th a mãn: f ( 2002 x − f ( 0 ) ) = 2002 x 2 ∀x ∈ R . 1  1  4) Tìm f : R { 0 } → R th a mãn: f ( x ) + f  = 1 ∀x ∈ R { 0;1} . 2x  1 − x  1− x  5) Tìm f : R { ± 1; 0} → R th a mãn: ( f ( x ) ) f   = 64 x ∀x ∈ R {−1} .  1+ x  2  2x  2 6) Tìm f : R   → R th a mãn: 2 f ( x ) + f  = 996 x ∀x ≠ . 3  3x − 2  3  x −3  x+3 7) Tìm f : R { ± 1 } → R th a mãn: f  + f  = x ∀x ≠ ±1 .  x +1   1− x  8) Tìm f : R → R th a mãn: 2 f ( x ) + f (1 − x ) = x 2 ∀x ∈ R . 1 9) Tìm f : R → R th a mãn: f ( x ) + f   = x 2008 ∀x ∈ R* . x  1  x −1  1 10) Tìm f : R ±  → R th a mãn: f ( x ) + f   = x ∀x ≠ .  3  1 − 3x  3  a2  11) Tìm f : R → R th a mãn: f ( x ) + f   = x ∀x ≠ a ( a > 0) . a−x f ( 2 x + 1) + 2 g ( 2 x + 1) = 2 x  12) Tìm f , g : R { 1 } → R th a mãn:   x   x  ∀x ≠ 1 . f  + g =x   x −1   x −1  Phương pháp 3: Phương pháp chuy n qua gi i h n.  2 x  3x Ví d 1: Tìm hàm s f : R → R liên t c, th a mãn: f ( x ) + f   = ∀x ∈ R (1) .  3  5 L i gi i: 2x 3 ð t x1 = ; (1) ⇒ f ( x ) + f ( x1 ) = x . 3 5 2 x1 3 ð t x2 = ; (1) ⇒ f ( x1 ) + f ( x2 ) = x1 . 3 5 8
  • 9. 2 xn 3 ð t xn +1 = , n ∈ N * ; (1) ⇒ f ( xn ) + f ( xn +1 ) = xn . 3 5  3  f ( x ) + f ( x1 ) = 5 x (1)   f (x )+ f (x ) = 3 x  ( 2) 1 2 1 Ta có h  5 ……  f x + f x 3  ( n ) ( n+1 ) = xn ( n + 1)  5 Nhân dòng phương trình th (i) v i (-1)i+1 r i c ng l i ta ñư c: 3  2  2  2  2 n n+2 f ( x ) + ( −1) f ( xn +1 ) = x 1 − +   − ⋯ +  −   ( *) . 5  3  3   3   ( f l.tôc ) Xét lim ( −1) f ( xn +1 )  = lim  f ( xn +1 )  = n+ 2   f ( lim xn +1 ) = f ( 0 ) .   n+ 2 M t khác (1) suy ra f(0) = 0 nên lim ( −1) f ( xn +1 ) = 0 . 3 1 9x L y gi i h n hai v c a (*) ta ñư c: f ( x ) = x = . Th l i th y ñúng. 5 1 + 2 25 3 9x V y hàm s c n tìm là: f ( x ) = . 25 Ví d 2: Tìm hàm s f liên t c t i xo= 0 th a mãn: f : R → R và 2 f ( 2 x ) = f ( x ) + x ∀x ∈ R ( 2) . L i gi i: t t ð t t = 2 x ta ñư c: 2 f ( t ) = f   + ∀t ∈ R ( 2' ) . 2 2  1 * tn +1 = 2 tn , ∀n ∈ N  Xét dãy:  . Thay dãy {tn} vào (2’) ta ñư c: t = 1 t 1 2   1 1  f ( t ) = 2 f ( t1 ) + 4 t (1)   f (t ) = 1 f (t ) + 1 t  1 ( 2 ) . Th  2 2 4 1 (n) vào ( n − 1) → ( n − 2 ) → ⋯ ta ñư c: ⋯⋯  f t 1 1  ( n −1 ) 2 ( n ) 4 n −1 (n) = f t + t  1 1 1 1 f (t ) = 2 n f ( tn ) + n +1 f ( tn −1 ) + n f ( tn − 2 ) + ⋯ + 2 t 2 2 2 (* ) . ' 9
  • 10. n 1 1  1 1 1  Thay tn =   t vào (*’) ta ñư c: f ( t ) = n f ( tn ) + t  2 + 4 + ⋯ + 2 n  2 2 2 2 2  (* ) . "  1  t Vì f liên t c t i xo = 0 nên lim  n f ( tn )  = 0 . L y gi i h n 2 v (*”) suy ra: f ( t ) = . Th 2  3 l i th y ñúng. Nh n xét: +) N u dãy {xn} tu n hoàn thì ta gi i theo phương pháp th r i quy v h pt hàm. +) N u dãy {xn} không tu n hoàn nhưng f liên t c t i xo = 0 và {xn} → 0 thì s d ng gi i h n như VD1. + N u {xn} không tu n hoàn, không có gi i h n thì ph i ñ i bi n ñ có dãy {tn} có gi i h n 0 và làm như ví d 1. BÀI T P 1) Tìm f : R → R th a mãn: a) f liên t c t i xo = 0, b) n f ( nx ) = f ( x ) + nx ∀n ∈ N , n ≥ 2; ∀x ∈ R .  x  10 2) Tìm f : R → R liên t c t i xo = 0, th a mãn: f ( 3 x ) + f   = x . 3 3 3) Tìm f : R → R liên t c t i xo = 0, th a mãn: m f ( mx ) − n f ( nx ) = ( m + n ) x ∀m, n ∈ N * , m ≠ n , ∀x ∈ R . Phương pháp 4: Phương pháp xét giá tr . +) ðây là phương pháp cơ s c a m i phương pháp khác. +) Khi v n d ng phương pháp c n chú ý s d ng k t qu v a có ñư c. ( a ) f ( x ) ≥ 0 ∀x ∈ R  Ví d 1: Tìm f : R → R th a mãn:  . ( b ) f ( x + y ) ≥ f ( x ) + f ( y ) ∀x, y ∈ R  L i gi i: x = 0  f ( 0) ≥ 0  Cho  suy ra  ⇒ f (0) = 0 . y = 0  f ( 0) ≥ 2 f ( 0)   f ( 0) ≥ f ( x ) + f ( − x )  f ( x) + f ( − x ) ≤ 0   Cho y = − x ⇒  ⇒  f ( x ) ≥ 0, f ( − x ) ≥ 0   f ( x ) ≥ 0, f ( − x ) ≥ 0  ⇒ f ( x ) = f ( − x ) = 0 ∀x ∈ R . V y f ( x ) = 0 . Th l i th y ñúng. Ví d 2: Tìm f : R → R th a mãn: 1 1 1 f ( xy ) + f ( yz ) − f ( x ) f ( yz ) ≥ ∀x, y, z ∈ R ( 2) . 2 2 4 L i gi i: 10
  • 11. 2 2 1  1 1 Cho x = z , y = 1 ta ñư c: f ( x ) − ( f ( x ) ) ≥ ⇔  f ( x ) −  ≤ 0 ⇔ f ( x ) = . Th l i th y 4  2 2 ñúng. Ví d 3: Tìm f : R → R th a mãn: f ( x ) = Max { xy − f ( y ) } ∀x ∈ R ( 3) . y∈R L i gi i: ( 3) ⇒ f ( x ) ≥ xy − f ( y ) ∀x, y ∈ R . t2 Cho x = y = t ∈ R ⇒ f ( t ) = ∀t ∈ R (a) . 2 T (a) suy ra: y2 x2 1 2 x2 x2 xy − f ( y ) ≤ xy − = − ( x − y) ≤ ⇒ f ( x ) = Max { xy − f ( y ) } ≤ ∀x ∈ R (b ) 2 2 2 2 y∈R 2 x2 ( a ) + (b) ⇒ f ( x) = . Th l i th y ñúng. 2 Ví d 4: Tìm f : R → R th a mãn: f ( x + y ) ≥ f ( x ) f ( y ) ≥ 2008x + y ∀x, y ∈ R ( 4) . L i gi i: 2 Cho x = y = 0 ⇒ f ( 0 ) ≥ ( f ( 0 ) ) ≥ 1 ⇒ f ( 0 ) = 1 . Cho 1 x = − y ∈ R ⇒ 1 = f ( 0 ) ≥ f ( x ) f ( − x ) ≥ 1⇒ f ( x ) f ( − x ) = 1⇒ f ( x ) = ∀x ∈ R (a) . f ( −x)  f ( x ) ≥ 2008 x > 0  Cho y = 0; x ∈ R ⇒ f ( x ) ≥ 2008 ⇒  x (b) .  f ( − x ) ≥ 2008 > 0 −x  1 1 Theo ( a ) + ( b ) ⇒ f ( x ) = ≤ = 2008x ( c ) . ( b ) + ( c ) ⇒ f ( x ) = 2008x . Th l i f ( − x ) 2008− x th y ñúng. Ví d 5: Tìm f : [ a; b ] → [ a ; b ] th a mãn: f ( x ) − f ( y ) ≥ x − y ∀x, y ∈ [ a ; b ] (a < b cho trư c) (5). L i gi i: Cho x = a ; y = b ⇒ f ( a ) − f ( b ) ≥ a − b = b − a ( a ) . vì f ( a ) , f ( b )∈ [ a ; b ] nên f ( a ) − f ( b ) ≤ a − b = b − a ( b ) . 11
  • 12.  f  (a) = a   f (b) = b ( a ) + ( b ) ⇒ f ( a ) − f ( b ) = b − a ⇔  .   f (a) = b  f   (b) = a  f (a) = a  +) N u  thì:  f (b) = b  Ch n y = b ; x ∈ [ a ; b ] ⇒ f ( x ) ≤ x ( c ) . Ch n y = a ; x ∈ [ a ; b ] ⇒ f ( x ) ≥ x ( d ) . (c) + (d ) ⇒ f ( x) = x .  f (a) = b  +) N u  thì:  f (b) = a  Ch n y = b ; x ∈ [ a ; b ] r i ch n y = a ; x ∈ [ a ; b ] như trên ta ñư c: f ( x ) = a + b − x . Th l i th y ñúng. Nh n xét: +) T VD1 → VD5 là các BPT hàm. Cách gi i nói chung là tìm các giá tr ñ c bi t – có th tính ñư c trư c. Sau ñó t o ra các BðT “ngư c nhau” v hàm s c n tìm ñ ñưa ra k t lu n v hàm s . +) Vi c ch n các trư ng h p c a bi n ph i có tính “k th a”. T c là cái ch n sau ph i d a vào cái ch n trư c nó và th các kh năng có th s d ng k t qu v a có ñư c. Ví d 6: Tìm f : R → R th a mãn:  π   f ( 0 ) = a ; f   = b ( a, b cho tr−íc )  2 (6) .  f ( x + y ) + f ( x − y ) = 2 f ( x ) cos y ∀x, y ∈ R  L i gi i: π  π  π Cho y = ; x ∈ R ta ñư c: f  x +  + f  x −  = 0 (a) . 2  2  2 Cho x = 0; y ∈ R ta ñư c: f ( y ) + f ( − y ) = 2a cos y (b) . π π  π  Cho x = ; y ∈ R ta ñư c: f  + y  + f  − y  = 2b cos y (c) . 2 2  2  12
  • 13.  π  π f x+ + f x− =0   2  2   π π   π ( a ) + (b) + ( c ) ⇒  f x− + f  − x  = 2a cos  x −  .   2 2   2   π π  f  x +  + f  − x  = 2b cos x   2 2  Gi i h ta ñư c: f ( x ) = a cos x + b sin x . Th l i th y ñúng. Ví d 7: Tìm f : R → R th a mãn: f ( x ) f ( y ) = f ( x + y ) + sin x sin y ∀x, y ∈ R (7) . L i gi i: Ta th y f ( x ) = cos x là m t hàm s th a mãn. 2  f (0) = 0 Cho x = y = 0 ⇔ ( f ( 0 ) ) = f ( 0 ) ⇔  .  f (0) = 1  N u f ( 0 ) = 0 thì: Cho y = 0; x ∈ R ⇒ f ( x ) = − f ( 0 ) = 0 ∀x ∈ R . Th l i ta ñư c: sin x sin y = 0 ∀x, y ∈ R ⇒ vô lý. V y f ( x ) = 0 không là nghi m (7). N u f ( 0 ) = 1 thì cho x = − y ⇒ f ( x ) f ( − x ) = 1 + ( − sin 2 x ) = cos 2 x ⇒ f ( x ) f ( − x ) = cos 2 x ( a ) .  π  f  =0 π 2 Cho x = ⇒  . 2   π f −  = 0   2 π  π N u f   = 0 thì: Cho x = ; y ∈ R th vào (7) suy ra: 2 2  π f  y +  + sin y = 0 ⇒ f ( y ) = cos y ∀y ∈ R . Th l i th y ñúng.  2  π N u f  −  = 0 tương t như trên ta ñư c: f ( y ) = cos y ∀y ∈ R .  2 V y hàm s c n tìm là: f ( x ) = cos x . Ví d 8: Tìm f , g : R → R th a mãn: f ( x ) − f ( y ) = cos ( x + y ) g ( x − y ) ∀x, y ∈ R ( 8) . L i gi i: π π  π  Ch n x = − y; y∈ R (8) ⇒ f  − y  − f ( y) = 0 ⇔ f  − y  = f ( y) (a) . 2  2   2  π π  π  Ch n x = + y ; y ∈ R ( 8 ) ⇒ f  + y  − f ( y ) = − sin 2 y.g   ( b ) . 2 2  2 13
  • 14. π  π  π  ( a ) + (b) ⇒ f  + y− f  − y  = − sin 2 y. g   ( c ) . 2  2  2 π  π  Theo (8): f  + y  − f  − y  = − g (2y) (d ) . 2  2  ( c ) + ( d ) ⇒ g ( 2 y ) = sin 2 y. g  π   ∀y ∈ R ⇒ g ( 2 x ) = a sin 2 x ⇒ g ( x ) = a sin x ∀x ∈ R . 2 π  (v i a = g   cho trư c.) 2 a Cho y = 0; x ∈ R ⇒ f ( x ) − f ( 0 ) = cos x. g ( x ) ⇒ f ( x ) = sin 2 x + b (b = f ( 0 )) , ∀x ∈ R . 2  a  f ( x ) = sin 2 x + b Th l i 2 hàm s :  2 (V i a, b là h ng s cho trư c). Th a mãn (8).  g ( x ) = a sin x   f ( − x ) = − f ( x ) ∀x ∈ R ( a )   Ví d 9: Tìm f : R → R th a mãn:  f ( x + 1) = f ( x ) + 1 ∀x ∈ R ( b ) .  f  1  f ( x)    = 2 ∀x ≠ 0 ( c )   x x L i gi i:  x +1  Ta tính f   ñ n f ( x ) theo hai cách:  x   x +1   1 1 f ( x) f  = f 1 +  = 1 + f   = 1 + 2 ∀x ≠ 0 ( a ) .  x   x x x  x   1  f  f 1 −  2  x +1  x +1   x +1    1  =  x +1 f  2 =  2 =  1 + f −  =  x   x   x   x    x +1       x +1  x +1   x +1 2    1     x +1  2  f ( x + 1)  =  1 +  − f   =   1 − =  x     x +1    x   ( x + 1)  2    x +1  2  1+ f ( x)    1 −  ∀x ≠ 0, x ≠ 1 ( b ) .  x   ( x + 1)  2   ( a ) + ( b ) ⇒ f ( x ) = x ∀x ≠ 0; x ≠ 1 . V i x = 0; ( a ) ⇒ f ( 0 ) = 0 th a mãn f ( x ) = x . V i x = 1; ( a ) ⇒ f ( −1) = − f (1) : Cho x = 0; ( b ) ⇒ f (1) = 1 ⇒ f ( −1) = −1 th a mãn f ( x ) = x . 14
  • 15. V y f ( x ) = x ∀x ∈ R . Th l i th y ñúng . Ví d 10: Tìm f : R { 0 } → R th a mãn:  f (1) = 1 ( a )    1  1 1 f   = f   . f   ∀x, y ≠ 0 ( b ) .   x+ y x  y ( x + y ) f ( x + y ) = xy f ( x ) f ( y ) ∀x, y tháa m n xy ( x + y ) ≠ 0 ( c )  L i gi i:  1  1 Cho x = y ∈ R* , ( b ) ta ñư c: f   = 2 f   ⇒ f ( x ) = 2 f ( 2 x ) ∀x ≠ 0 (*)  2x   x 2 2 Cho x = y ∈ R* , ( c ) ta ñư c: 2 x f ( 2 x ) = x 2 ( f ( x ) ) ⇔ 2 f ( 2 x ) = x ( f ( x ) ) ∀x ≠ 0 (*' ) . 2 Th (*) vào (*’) suy ra: f ( x ) = x ( f ( x ) ) (* ) . " Gi s : ∃ xo ≠ 1, xo ∈ R* sao cho: f(xo) = 0. Thay x = 1 − xo ; y = xo vào (*”) ta ñư c: f(1) = 0 trái v i gi thi t f(1) = 1. V y f ( x ) ≠ 0 ∀x ≠ 1; x ≠ 0 . 1 Vì f (1) = 1 ≠ 0 nên t (*”) suy ra f ( x ) = ∀x ≠ 0 . Th l i th y ñúng. x Ví d 11: Tìm f : R → R th a mãn:  f (1) = 1 ( a )   f ( x + y ) = f ( x ) + f ( y ) + 2 xy ∀x, y ∈ R ( b ) .  f  1  f ( x)    = 4 ∀x ≠ 0 ( c )  x x L i gi i: Cho x = y = 0, ( b ) ⇔ f ( 0 ) = 0 Cho x = y = t ≠ 0, ( b ) ⇔ f ( 2t ) − 2 f ( t ) = 2t 2 (1) . 1 1 1 1 Cho x = y = , (b) ⇔ f   − 2 f   = 2 ( *) 2t t  2t  2t 1  f (t )  1  f ( 2t ) f (t ) f ( 2t ) (c) ⇒ f  1 T  = 4 ; f  = 4 . Th vào (*) ta ñư c: 4 − 2 4 = 2 ( 2) . t  t  2t  ( 2t ) t ( 2t ) 2t (1) + ( 2 ) ⇒ f ( t ) = t 2 ∀t ≠ 0 . T f ( 0 ) = 0 ⇒ f ( t ) = t 2 ∀t ∈ R . Th l i th y ñúng. Ví d 12: Cho hàm s f : ( 0; + ∞ ) → ( 0; + ∞ ) th a mãn:  f ( x)  f  = y f ( y ) f ( f ( x ) ) ∀x, y ∈( 0; + ∞ ) (12 ) .  y  15
  • 16. L i gi i: Cho: x = y = 1 ⇒ f ( f (1) ) = f (1) . f ( f (1) ) ⇒ f (1) = 1 vì f ( f (1) ) ≠ 0 ⇒ f ( f (1) ) = 1 . 1 f   f (1)   y a . x = 1; y ∈ ( 0; + ∞ ) ⇒ f   = y f ( y ) f ( f (1) ) = y f ( y ) ⇔ f ( y ) = ( )  y  y M t  1     f    f ( y)    1  khác: f ( f ( y ) ) = f     = y f ( y ) f  f    = y f ( y ) f ( y f ( y ) ) = y f ( y ) y f   y    y   1       y      1 1 = y f ( y) f   f ( f ( y )) . y  y 1 1 1 Vì f ( f ( y ) ) ≠ 0 nên y f ( y ) f   = 1 ⇔ f ( y) f   = 1 (b) . y  y  y 1 ( a ) + (b) ⇒ f ( y ) = ∀y ∈( 0; + ∞ ) . Th l i th y ñúng. y Ví d 13: Tìm f : R → R th a mãn:  1  f ( 0) = ( a )  2 . ∃ a ∈ R : f ( a − y ) f ( x ) + f ( a − x ) f ( y ) = f ( x + y ) ∀x, y ∈ R ( b )  L i gi i: 1 Cho x = y = 0, ( b ) ⇒ f ( a ) = . 2 Cho y = 0; x ∈ R ta ñư c: f ( x ) = f ( x ) . f ( a ) + f ( 0 ) . f ( a − x ) ⇒ f ( x ) = f ( a − x ) ( c ) . 2 2 Cho y = a − x ; x ∈ R ta ñư c: f ( a ) = ( f ( x ) ) + ( f ( a − x ) ) (d ) .  1 2 1  f ( x) = 2 ( c ) + ( d ) ⇒ 2 ( f ( x )) = ⇔ . 2  f ( x) = − 1   2 1 N u ∃ xo ∈ R sao cho: f ( xo ) = − thì: 2 2 1 (b) x x  x   x  (c)   x   − = f ( xo ) = f  o + o  = 2 f  o  . f  a − o  = 2  f  o   ≥ 0 ⇒ Vô lí. 2  2 2  2  2   2  1 V y f ( x) = ∀x ∈ R . Th l i th y ñúng. 2 16
  • 17. Ví d 14: (VMO.1995) Tìm f : R → R th a mãn: f (( x − y ) ) = x 2 2 2 − 2 y f ( x ) + ( f ( y ) ) ∀x, y ∈ R (14 ) . L i gi i: 2  f ( 0) = 0 Cho x = y = 0 ⇒ f ( 0 ) = ( f ( 0 ) ) ⇔  .  f ( 0) = 1  y = 0 N u f ( 0 ) = 0 : Cho  ta ñư c: f ( x 2 ) = x 2 ⇒ f ( t ) = t ∀t ≥ 0 x ∈ R 2 2 Cho x = y ∈ R ta ñư c: f ( 0) = x2 − 2 x f ( x ) + ( f ( x ) ) ⇔ ( f ( x ) − x ) = 0 ⇔ f ( x ) = x . Th l i th y ñúng. y = 0 N u f ( 0 ) = 1: Cho  ta ñư c: f ( x 2 ) = x 2 + 1 ⇔ f ( t ) = t + 1 ∀t ≥ 0 .  x∈R 2 2 Cho x = 0; y ∈ R ta ñư c: f ( y 2 ) = −2 y + ( f ( y ) ) ⇒ ( f ( y ) ) = f ( y 2 ) + 2 y 2  f ( y) = y +1 = y 2 + 1 + 2 y = ( y + 1) ⇒  .  f ( y) = − y −1  Gi s ∃ yo ∈ R sao cho: f ( yo ) = − yo − 1 . Ch n x = y = yo ta ñư c: 2  f ( yo ) = yo − 1 1 = yo − 2 yo f ( yo ) + ( f ( yo ) ) ⇔  2 .  f ( yo ) = yo + 1  N u f ( yo ) = yo − 1 ⇒ − yo − 1 = yo − 1 ⇒ yo = 0 v f ( 0 ) = −1 (lo¹i) . N u f ( yo ) = yo + 1 ⇒ − yo − 1 = yo + 1 ⇒ yo = −1 ⇒ f ( −1) = 0 . Th a mãn: f ( yo ) = yo + 1 . V y f ( y ) = y + 1 ∀y ∈ R . Th l i th y ñúng. Ví d 15: (VMO.2005) Tìm f : R → R th a mãn: f ( f ( x − y ) ) = f ( x ) f ( y ) − f ( x ) + f ( y ) − xy ∀x, y ∈ R (15 ) . L i gi i: 2 Cho x = y = 0 ⇒ f ( f ( 0 ) ) = ( f ( 0 ) ) . ð t f ( 0 ) = a ⇒ f ( a ) = a 2 . 2 2 Cho x = y ∈ R ⇒ ( f ( x ) ) = x 2 + f ( a ) ⇒ ( f ( x ) ) = x 2 + a 2 (*) . 2 2  f ( x) = f (−x) ⇒ ( f ( x )) = ( f ( − x )) ⇒  .  f ( x) = − f (−x)  N u ∃ xo ∈ R* sao cho f ( xo ) = f ( − xo ) : + Ch n x = 0; y = − xo ⇒ f ( f ( xo ) ) = a f ( − xo ) − a + f ( − xo ) ( a ) . 17
  • 18. + Ch n y = 0; x = − xo ⇒ f ( f ( xo ) ) = a f ( xo ) + a − f ( xo ) ( b ) . ( a ) + ( b ) ⇒ a ( f ( xo ) − f ( − xo ) ) − ( f ( xo ) + f ( − xo ) ) + 2a = 0 ( c ) . (*) 2 Vì f ( xo ) = f ( − xo ) nên f ( xo ) = a ⇒ ( f ( xo ) ) = x0 + a 2 ⇒ a 2 = x0 + a 2 ⇒ xo = 0 trái v i 2 2 gi thi t xo ∈ R* . V y f ( x ) = − f ( − x ) ∀x ∈ R . Ta th y (c) không ph thu c vào xo nên ta có: a ( f ( x ) − f ( − x ) ) − ( f ( x ) + f ( − x ) ) + 2a = 0 ( c ) . Thay f ( x ) = − f ( − x ) suy ra: a = 0 a ( f ( x ) + 1) = 0 ⇔  .  f ( x ) = −1 (*) 2  f ( x) = x + N u a = 0 ⇒ ( f ( x )) = x2 ⇔  .  f ( x) = −x  Gi s t n t i xo ∈ R* ñ f ( xo ) = xo . Khi ñó (b) suy ra: xo = f ( xo ) = a xo + a − xo ⇒ xo = 0 trái gi thi t xo ∈ R* . V y f ( x ) = − x ∀x ∈ R . Th l i th y ñúng + N u f ( x ) = −1 ∀x ∈ R . Th l i ta ñư c (15 ) ⇔ xy = 2 ∀x, y ∈ R . Vô lí. V y hàm s c n tìm là: f ( x ) = − x . Nh n xét: Có m t suy lu n hay nh m l n ñư c s d ng các VD:   1   2 1  f ( x) = 2   2 2  f ( y ) = y +1  VD13  ( f ( x ) ) = ⇔   ; VD14  ( f ( y ) ) = ( y + 1) ⇔  ;  4  f ( x) = − 1    f ( y ) = − y − 1         2  2  f ( x) = x  VD15  ( f ( x ) ) = x 2 ⇔   , ñó là hi u sai:   f ( x) = −x      1 2 1  f ( x ) = 2 ∀x ∈ R ( f ( x )) = ⇔  ; 4  f ( x ) = − 1 ∀x ∈ R   2 2 2  f ( y ) = y + 1 ∀x ∈ R ( f ( y ) ) = ( y + 1) ⇔  ;  f ( y ) = − y − 1 ∀x ∈ R  2  f ( x ) = x ∀x ∈ R ( f ( x )) = x2 ⇔  .  f ( x ) = − x ∀x ∈ R  18
  • 19. 2 1 Th c t thư ng là như v y nhưng v m t logic thì không ñúng. ( f ( x ) ) = thì f ( x ) có th 4 1  1  2 ( x ≥ 0)  2 1  f ( x) = 2 là hàm khác n a như f ( x ) =  . Như v y ( f ( x )) = ⇔  ch − 1 ( x < 0 ) 4  f ( x) = − 1  2    2 1 ñúng v i m i x c th ch không th k t lu n ch có hai hàm s f ( x) = ∀x ∈ R ho c 2 1 f ( x) = − ∀x ∈ R . 2 1 1 ð gi i quy t v n ñ này ta thư ng “th ” f ( x ) = ∀x ∈ R ho c f ( x ) = − ∀x ∈ R vào ñ 2 2 1 bài ñ tìm hàm s không th a mãn (trong VD13 thì f ( x ) = không th a mãn) sau ñó l p 2 1 lu n ph ñ nh là ∃ xo : f ( xo ) = − ñ d n ñ n vô lí! 2 Ví d 16: Tìm f : (0,1) → ℝ th a mãn: f(xyz) = xf(x) + yf(y) +zf(z) ∀x, y , z ∈ (0,1) . L i gi i: Ch n x = y = z: f(x3) = 3xf(x). Thay x, y, z b i x2: f(x6) = 3 x2 f(x2). M t khác: f(x6) = f(x. x2 .x3) = xf(x) + x2 f(x2) + x3 f(x3). ⇒ 3 x2 f(x2) = xf(x) + x2 f(x2) + 3x4 f(x) ⇔ 2 x2 f(x2) = xf(x) + 3x4 f(x) 3x3 + 1 ⇒ f ( x2 ) = f ( x), ∀x ∈ ℝ 2 Thay x b i x3 ta ñư c : 3 x9 + 1 f ( x6 ) = f ( x 3 ), ∀x ∈ ℝ 2 3x9 + 1 ⇒ 3x 2 f ( x 2 ) = 3 xf ( x), ∀x ∈ ℝ 2 3x3 + 1 3x9 + 1 ⇒ 3x 2 f ( x) = 3 xf ( x), ∀x ∈ ℝ 2 2 ⇒ f ( x) = 0, ∀x ≠ 0 V y f(x) = 0 v i m i x ∈(0; 1). BÀI T P 5 1) Tìm f : N → R th a mãn: f ( 0 ) ≠ 0; f (1) = ; 2 f ( x ) f ( y ) = f ( x + y ) + f ( x − y ) ∀x, y ∈ N , x ≥ y . 2) Tìm f : N → R th a mãn: f ( m + n ) + f ( n − m ) = f ( 3n ) ∀m, n ∈ N , n ≥ m . 19
  • 20. 3) Tìm f : R → R th a mãn: f ( x f ( y ) ) = y f ( x ) x, y ∈ R . 4) Tìm f : R → R th a mãn: f ( ( x + 1) f ( y ) ) = y ( f ( x ) + 1) x, y ∈ R . 5) Tìm f : ( 0; + ∞ ) → ( 0; + ∞ ) th a mãn: f ( x ) = Max  x 2 y + y 2 x − f ( y )  ∀x ∈ ( 0; + ∞ ) . y∈( 0; +∞ )   6) Tìm f : R → R th a mãn: f ( xy ) − f ( x − y ) + f ( x + y + 1) = xy + 2 x + 1 ∀x, y ∈ R .  f ( xy ) = f ( x ) f ( y )  7) Tìm f : [ 1; + ∞ ) → [ 1; + ∞ ) th a mãn:  ∀x, y ∈ [ 1; + ∞ ) .  f ( f ( x )) = x  8) Tìm f : R → R th a mãn: f ( xy ) = f ( x ) f ( y ) − f ( x + y ) + 1 ∀x, y ∈ R . 9) Tìm f : R → R th a mãn: ( f ( x ) + f ( z ) ) ( f ( y ) + f ( t ) ) = f ( xy − zt ) + f ( xt + zy ) ∀x, y, z , t ∈ R . 10) Tìm f : R → R th a mãn: f ( x 2 − y 2 ) = x f ( y ) − y f ( x ) ∀x, y ∈ R . 11) Tìm f : N → [ 0; + ∞ ) th a mãn: 1 f (1) = 1; f ( m + n ) + f ( m − n ) = 2 ( f ( 2m ) + f ( 2n ) ) ∀m, n ∈ N , m ≥ n .  x + y  f ( x) + f ( y) 12) Tìm f : Z → R th a mãn: f  = ∀x, y ∈ Z ; ( x + y )⋮ 3 .  3  2 13) Tìm f : N → N th a mãn: 3 f ( n ) − 2 f ( f ( n ) ) = n ∀n ∈ N . 14) Tìm f : Z → Z th a mãn: f (1) = a ∈ Z ; f ( m + n ) + f ( m − n ) = 2 ( f ( m ) + f ( n ) ) ∀m, n ∈ Z . 15) Tìm f : R → R th a mãn: f ( x 3 + 2 y ) = f ( x + y ) + f ( 3 x + y ) + 1 ∀x, y ∈ R . 16) Tìm f : R → R th a mãn: x 2 f ( x ) + f (1 − x ) = 2 x − x 4 ∀x ∈ R . Phương pháp 4: S d ng tính ch t nghi m c a m t ña th c. Ví d 1: Tìm P(x) v i h s th c, th a mãn ñ ng th c: ( x3 + 3 x 2 + 3 x + 2) P( x − 1) = ( x 3 − 3 x 2 + 3 x − 2) P( x), ∀x (1) L i gi i: (1) ⇔ ( x + 2)( x 2 + x + 1) P ( x − 1) = ( x − 2)( x 2 − x + 1) P( x), ∀x Ch n: x = −2 ⇒ P ( −2) = 0 x = −1 ⇒ P(−1) = 0 x = 0 ⇒ P(0) = 0 x = 1 ⇒ P(1) = 0 V y: P(x) = x(x – 1)(x + 1)(x + 2)G(x). 20
  • 21. Thay P(x) vào (1) ta ñư c: ( x + 2)( x 2 + x + 1)( x − 1)( x − 2) x( x + 1)G ( x − 1) = ( x − 2)( x 2 − x + 1) x( x − 1)( x + 1)( x + 2)G ( x), ∀x ⇒ ( x 2 + x + 1) G ( x − 1) = ( x 2 − x + 1)G ( x), ∀x G ( x − 1) G ( x) ⇔ 2 = 2 , ∀x x − x +1 x + x +1 G ( x − 1) G ( x) ⇔ 2 = 2 , ∀x ( x − 1) + ( x − 1) + 1 x + x + 1 G ( x) ð t R( x) = 2 (x ≠ 0, ± 1, -2) x + x +1 ⇒ R ( x) = R ( x − 1) (x ≠ 0, ± 1, -2) ⇒ R( x) = C V y P( x) = C ( x 2 + x + 1) x( x − 1)( x + 1)( x + 2) Th l i th y P(x) th a mãn ñi u ki n bài toán. Chú ý: N u ta xét P(x) = (x3 + 1)(x – 1) thì P(x + 1) = (x3 + 3x2 + 3x + 2)x. Do ñó (x3 + 3x2 + 3x + 2)xP(x) = (x2 – 1)(x2 – x + 1)P(x + 1). T ñó ta có bài toán sau: Ví d 2: Tìm ña th c P(x) v i h s th c, th a mãn ñ ng th c: (x3 + 3x2 + 3x + 2)xP(x) = (x2 – 1)(x2 – x + 1)P(x + 1) v i m i x. Gi i quy t ví d này hoàn toàn không có gì khác so v i ví d 1. Tương t như trên n u ta xét: P(x) = (x2 + 1)(x2 – 3x + 2) thì ta s có bài toán sau: Ví d 3: Tìm ña th c P(x) v i h s th c th a mãn ñ ng th c: (4 x 2 + 4 x + 2)(4 x 2 − 2 x ) P ( x) = ( x 2 + 1)( x 2 − 3 x + 2) P(2 x + 1), ∀x ∈ ℝ Các b n có th theo phương pháp này mà t sáng tác ra các ñ toán cho riêng mình. Phương pháp 5: S d ng phương pháp sai phân ñ gi i phương trình hàm. 1. ð nh nghĩa sai phân: Xét hàm x(n) = xn: Sai phân c p 1 c a hàm xn là: △ xn = xn+1 − xn Sai phân câp 2 c a hàm xn là: △2 xn =△ xn +1 −△ xn = xn + 2 − 2 xn +1 + xn k Sai phân câp k c a hàm xn là: △k xn = ∑ (−1)i Cki xn + k −i i =0 2. Các tính ch t c a sai phân: +) Sai phân các c p ñ u ñư c bi u th qua các giá tr hàm s . +) Sai phân có tính tuy n tính: ∆ k (af + bg ) = a∆ k f + b∆ k g +) N u xn ña th c b c m thì ∆ k xn : Là ña th c b c m – k n u m > k. Là h ng s n u m = k. Là 0 n u m < k. 21
  • 22. 3. N i dung c a phương pháp này là chuy n bài toán phương trình hàm sang bài toán dãy s và dùng các ki n th c dãy s ñ tìm ra các hàm s c n tìm. Ví d 1: Tìm f: N → R tho mãn : f(1) = 1 và 2f(n).f(n+k) = 2f(k-n) + 3f(n).f(k) ∀ k, n∈N, k≥ n. L i gi i:  f (0) = 0 Cho n = k = 0 ta ñư c: (f(0))2 + 2f(0) = 0 ⇔  .  f (0) = −2 + N u f(0) = 0 thì ch n n = 0, k∈ N ta ñư c: f(k) = 0 trái gi thi t f(1) = 1. + N u f(0) = - 2 thì ch n n = 1, k∈ N* ta ñư c: 2.f(k+1) - 3.f(k) - 2.f(k-1) = 0. u0 = −2; u1 = 1  ð t uk = f(k) ta ñư c dãy s :  * . 2uk +1 − 3uk − 2uk −1 = 0 ∀k ∈ ℕ  1 T ñây tìm ñư c uk = f(k) = −2.(− ) k ∀k ∈ N . Th l i th y ñúng. 2 Ví d 2 (D tuy n IMO 1992): Cho a, b> 0. Tìm f: [0; +∞) → [0; +∞) tho mãn : f(f(x))+a.f(x) = b.(a+b).x ∀x∈ [0; +∞) (2) L i gi i: C ñ nh x∈ [0; +∞) và ñ t u0 = x, u1 = f(x), un+1 = f(un). T (2) ta ñư c : un+2 + a.un+1 - b.(a + b).un = 0. Gi i dãy s trên ta ñư c: un = c1.bn + c2.(-a -b)n (*). un b n b Vì un ≥ 0 ∀n∈N nên ta có: 0 ≤ n = c1.( ) + c2 .(−1) n . M t khác: 0 < < 1 nên ( a + b) a+b a+b b n un lim ( ) = 0 . Do ñó, n u c2 > 0 thì khi n l và n ñ l n thì < 0 vô lí !; còn n u n →+∞ a+b ( a + b) n un c2 < 0 thì khi n ch n và n ñ l n thì < 0 vô lí !. V y c2 = 0. Thay vào (*) ta ñư c ( a + b) n un = c1.bn. T u0 = x suy ra c1 = x và f(x) = bx. Do ñó f(x) = bx ∀x∈[0;+∞). Th l i th y ñúng. Ví d 3: Tìm t t c các hàm f : ℝ → ℝ th a mãn: f(f(x)) = 3f(x) – 2x , ∀x ∈ ℝ L i gi i : Thay x b i f(x) ta ñư c: f(f(f(x))) = 3f(f(x)) – 2f(x) , ∀x ∈ ℝ ……………………….. f (... f ( x)) = 3 f (... f ( x)) − 2 f (... f ( x)) n+ 2 n +1 n Hay f n+ 2 ( x) = 3 f n+1 ( x) − 2 f n ( x), n ≥ 0 ð t xn = f n ( x ), n ≥ 0 ta ñư c phương trình sai phân: xn+ 2 = 3 xn +1 − 2 xn Phương trình ñ c trưng là: λ 2 − 3λ + 2 = 0 ⇔ λ = 1 ∨ λ = 2 22
  • 23. V y xn = c1 + c2 2 n  x0 = c1 + c2 = x Ta có:   x1 = c1 + 2c2 = f ( x ) T ñó ta ñư c c1 = 2 x − f ( x), c2 = f ( x) − x V y f ( x) = x + c2 ho c f ( x) = 2 x − c1 Phương pháp 6: Phương pháp s d ng ánh x . Ví d 1: Tìm f: N*→ N* tho mãn: f(f(n)+m) = n+f(m+2007) ∀ m, n∈N* (1). L i gi i: Trư c h t ta ch ng minh f là ñơn ánh. Th t v y: f(n1) = f(n2) ⇒ f(f(n1)+1) = f(f(n2)+1) ⇒ n1 + f(1+2007) = n2 + f(1+2007) ⇒ n1 = n2. V y f là ñơn ánh. M t khác t (1) suy ra: ∀ m, n ∈ N*, f(f(n) + f(1)) = n + f(f(1) + 2007) ⇒ f(f(n)+f(1)) = n + 1 + f(2007+2007) = f(f(n+1)+2007). Vì f là ñơn ánh nên ta có: f(n) + f(1) = f(n+1) + 2007 ⇒ f(n+1) - f(n) = f(1) - 2007. ð t f(1) - 2007 = a. Khi ñó ta có f(n) = n.a + 2007. Thay l i (10) ta ñư c a2n = n ∀n∈N* ⇒ a2 = 1 ⇒ a = 1 ⇒ f(n) = n+2007. Ví d 2: Tìm f: R → R tho mãn: f(xf(x)+f(y)) = (f(x))2+y ∀ x, y∈R (2). L i gi i: D ràng ch ng minh f là ñơn ánh. M t khác, c ñ nh x thì ∀t∈R t n t i y = t - (f(x))2 ñ f(xf(x) + f(y)) = t. V y f là toàn ánh, do ñó f là song ánh. Suy ra t n t i duy nh t a∈R sao cho f(a) = 0. Cho x = y = a ta ñư c f(0) = a. Cho x = 0, y = a ta ñư c f(0) = a2 + a. V y a = a2 + a hay a = 0 ⇒ f(0) = 0. Cho x = 0, y∈R ta ñư c f(f(y)) = y (a). Cho y = 0, x∈R ta ñư c f(x.f(x)) = (f(x))2 ⇒ f(f(x).f(f(x))) = (f(f(x)))2. Theo (a) ta ñư c f(f(x).x)) = x2 ⇒ (f(x))2 = x2 ⇒ f(x) = x ho c f(x) = -x. Gi s t n t i a, b∈R* ñ f(a) = a, f(b) = -b. Khi ñó thay x = a, y = b thì t (2) suy ra: f(a2 - b) = a2 + b. Mà (a2 + b)2 ≠ (a2 - b)2 v i a, b∈ R* trái v i kh ng ñ nh (f(x))2 = x2. V y có hai hàm s là f(x) = x, ∀x∈R ho c f(x) = -x ∀x∈R. Th l i th y ñúng. Phương pháp 7: phương pháp ñi m b t ñ ng. 1. ð c trưng c a hàm: Như ta ñã bi t, phương trình hàm là m t phương trình thông thư ng mà nghi m c a nó là hàm. ð gi i quy t t t v n ñ này, c n phân bi t tính ch t hàm v i ñ c trưng hàm. Nh ng tính ch t quan tr c ñư c t ñ i s sang hàm s , ñư c g i là nh ng ñ c trưng hàm. +) Hàm tuy n tính f(x) = ax , khi ñó f(x + y) = f(x) + f(y). V y ñ c trưng hàm tuy n tính là: f(x + y) = f(x) + f(y) v i m i x, y. 23
  • 24. x+ y +) Hàm b c nh t f(x) = ax + b, khi ñó f(x) + f(y) = 2 f ( ) . V y ñ c trưng hàm ñây là 2  x + y  f ( x) + f ( y ) f = , ∀x, y ∈ ℝ  2  2 ð n ñây thì ta có th nêu ra câu h i là: Nh ng hàm nào có tính ch t f ( x + y ) = f ( x ) + f ( y ), ∀x, y ∈ ℝ . Gi i quy t v n ñ ñó chính là d n ñ n phương trình hàm. V y phương trình hàm là phương trình sinh b i ñ c trưng hàm cho trư c. +) Hàm lũy th a f ( x) = x k , x > 0 ð c trưng là f(xy) = f(x)f(y). +) Hàm mũ f ( x) = a x (a > 0, a ≠ 1) ð c trưng hàm là f(x + y) = f(x)f(y), ∀x, y ∈ ℝ +) Hàm Lôgarit f ( x) = log a x (a>0,a ≠ 1) ð c trưng hàm là f(xy) = f(x) + f(y). +) f(x) = cosx có ñ c trưng hàm là f(x + y) + f(x – y) = 2f(x)f(y). Hoàn toàn tương t ta có th tìm ñư c các ñ c trưng hàm c a các hàm s f(x) =sinx, f(x) = tanx và v i các hàm Hypebolic: e x − e− x +) Sin hypebolic shx = 2 e x + e− x +) cos hypebolic chx = 2 shx e x − e − x +) tan hypebolic thx = = chx e x + e − x chx e x + e − x +) cot hypebolic cothx = = shx e x − e − x +) shx có TXð là ℝ t p giá tr là ℝ chx có TXð là ℝ t p giá tr là [1, +∞ ) thx có TXð là ℝ t p giá tr là (-1,1) cothx có TXð là ℝ {0} t p giá tr là (−∞, −1) ∪ (1, +∞ ) Ngoài ra b n ñ c có th xem thêm các công th c liên h gi a các hàm hypebolic, ñ th c a các hàm hypebolic. 2. ði m b t ñ ng: Trong s h c, gi i tích, các khái ni m v ñi m b t ñ ng, ñi m c ñ nh r t quan tr ng và nó ñư c trình bày r t ch t ch thông qua m t h th ng lý thuy t. ñây, tôi ch nêu ng d ng c a nó qua m t s bài toán v phương trình hàm. Ví d 1: Xác ñ nh các hàm f(x) sao cho: f(x+1) = f(x) + 2 ∀x ∈ ℝ. L i gi i: Ta suy nghĩ như sau: T gi thi t ta suy ra c = c + 2 do ñó c = ∞ Vì v y ta coi 2 như là f(1) ta ñư c f(x + 1) = f(x) + f(1) (*) Như v y ta ñã chuy n phép c ng ra phép c ng. D a vào ñ c trưng hàm, ta ph i tìm a: f(x) = ax ñ kh s 2. Ta ñư c (*) ⇔ a ( x + 1) = ax + 2 ⇔ a = 2 V y ta làm như sau: ð t f(x) = 2x + g(x). Thay vào (*) ta ñư c: 24
  • 25. 2(x + 1) + g(x + 1) = 2x + g(x) + 2, ∀x ∈ ℝ ði u này tương ñương v i g(x + 1) = g(x), ∀x ∈ ℝ V y g(x) là hàm tu n hoàn v i chu kì 1. ðáp s f(x) = 2x + g(x) v i g(x) là hàm tu n hoàn v i chu kì 1. Nh n xét: Qua ví d 1, ta có th t ng quát ví d này, là tìm hàm f(x) th a mãn: f(x + a) = f(x) + b, ∀x ∈ ℝ , a, b tùy ý. Ví d 2: Tìm hàm f(x) sao cho: f(x + 1) = - f(x) + 2, ∀x ∈ ℝ (2). L i gi i: ta cũng ñưa ñ n c = -c + 2 do ñó c = 1. v y ñ t f(x) = 1 + g(x), thay vào (2) ta ñư c phương trình: g(x + 1) = - g(x), ∀x ∈ ℝ  1  g ( x + 1) = − g ( x)  g ( x ) = [ g ( x) − g ( x + 1)] Do ñó ta có:  ⇔ 2 ∀x ∈ ℝ (3).  g ( x + 2) = g ( x)  g ( x + 2) = g ( x)  1 Ta ch ng minh m i nghi m c a (3) có d ng: g ( x) = [ h( x) − h( x + 1) ] , ∀x ∈ ℝ ñó h(x) là 2 hàm tu n hoàn v i chu kì 2. Nh n xét: Qua ví d này, ta có th t ng quát thành: f(x + a) = - f(x) + b, ∀x ∈ ℝ , a, b tùy ý. Ví d 3: Tìm hàm f(x) th a mãn: f(x + 1) = 3f(x) + 2, ∀x ∈ ℝ (3). Gi i: Ta ñi tìm c sao cho c = 3c + 2 d th y c = -1. ð t f(x) = -1 + g(x). Lúc ñó (3) có d ng: g(x + 1) = 3g(x) ∀x ∈ ℝ Coi 3 như g(1) ta ñư c: g(x + 1) = g(1).g(x) ∀x ∈ ℝ (*). T ñ c trưng hàm, chuy n phép c ng v phép nhân, ta th y ph i s d ng hàm mũ: a x +1 = 3a x ⇔ a = 3 V y ta ñ t: g ( x) = 3x h( x) thay vào (*) ta ñư c: h(x + 1) = h(x) ∀x ∈ ℝ . V y h(x) là hàm tu n hoàn chu kì 1. K t lu n: f ( x) = −1 + 3x h( x) v i h(x) là hàm tu n hoàn chu kì 1. Nh n xét: ví d 3 này, phương trình t ng quát c a lo i này là: f(x + a) = bf(x) + c ∀x ∈ ℝ ; a, b, c tùy ý. +) V i 0< b ≠ 1: chuy n v hàm tu n hoàn. +) V i 0< b ≠ 1: chuy n v hàm ph n tu n hoàn. Ví d 4: Tìm hàm f(x) th a mãn f(2x + 1) = 3f(x) – 2 ∀x ∈ ℝ (4) Gi i: Ta có: c = 3c – 2 suy ra c = 1. ð t f(x) = 1 + g(x). Khi ñó (4) có d ng: g(2x + 1) = 3g(x) ∀x ∈ ℝ (*) Khi bi u th c bên trong có nghi m ≠ ∞ thì ta ph i x lý cách khác. T 2x + 1 = x suy ra x = 1. V y ñ t x = -1 + t ta có 2x + 1 = -1 + 2t. Khi ñó (*) có d ng: g(-1 + 2t) = 3g(-1 + t ) ∀t ∈ ℝ . ð t h(t) = g(-1 + 2t), ta ñư c h(2t) = 3h(t) (**). Xét 2t = t ⇔ t = 0 , (2t ) m = 3.t m ⇔ m = log 2 3 Xét ba kh năng sau: 25
  • 26. +) N u t = 0 ta có h(0) = 0. +) N u t> 0 ñ t h(t ) = t log2 3ϕ (t ) thay vào (3) ta có: ϕ (2t ) = ϕ (t ), ∀t > 0 . ð n ñây ta ñưa v ví d hàm tu n hoàn nhân tính. ϕ (2t ) = −ϕ (t ), ∀t < 0 +) N u t < 0 ñ t h(t ) =| t |log2 3 ϕ (t ) thay vào (3) ta ñư c ⇔  ϕ (4t ) = ϕ (t ), ∀t < 0  1 ϕ (t ) = [ϕ (t ) − ϕ (2t )] , ∀t < 0 ⇔ 2 . ϕ (4t ) = ϕ (t ), ∀t < 0  Nh n xét: Bài toán t ng quát c a d ng này như sau: f (α x + β ) = f (ax ) + b α ≠ 0, ± 1 . Khi ñó t phương trình α x + β = x ta chuy n ñi m b t ñ ng v 0, thì ta ñư c hàm tu n hoàn nhân tính. +) N u a = 0 bài toán bình thư ng. +) N u a = 1 ch ng h n xét bài toán sau: “Tìm f(x) sao cho f(2x + 1) = f(x) – 2, ∀x ≠ -1 (1)”. Xét: 2x + 1 = x ⇔ x = −1 nên ñ t x = -1 + t thay vào (1) ta ñư c: f(-1 + 2t) = f(-1 + t) + 2, ∀t ≠ 0 . ð t g(t) = f( - 1 + t) ta ñư c: g(2t) = g(t) + 2 ∀t ≠ 0 (2). T tích chuy n thành t ng nên là hàm logarit. 1 Ta có log a (2t ) = log a t − 2 ⇔ a = . V y ñ t g (t ) = log 1 t + h(t ) . Thay vào (2) ta có 2 2 h(2t ) = h(t ), ∀t ≠ 0 . ð n ñây bài toán tr nên ñơn gi n. Phương pháp 8: phương pháp s d ng h ñ m. Ta quy ư c ghi m = (bibi-1...b1)k nghĩa là trong h ñ m cơ s k thì m b ng bibi-1...b1. Ví d 1 (Trích IMO năm 1988): Tìm f: N*→ N* tho mãn: f(1) = 1, f(3) = 3, f(2n) = f(n), f(4n+1) = 2f(2n+1) - f(n); f(4n+3) = 3f(2n+1) - 2f(n) ∀n∈N* (12). L i gi i: Tính m t s giá tr c a hàm s và chuy n sang cơ s 2 ta có th d ñoán ñư c: “∀n∈N*, n = (bibi-1...b1)2 thì f(n) = (b1b2 ...bi)2” (*). Ta s ch ng minh (*) b ng quy n p. + V i n = 1, 2, 3, 4 d ki m tra (*) là ñúng. + Gi s (*) ñúng cho k < n, ta s ch ng minh (*) ñúng cho n (v i n ≥ 4). Th t v y, ta xét các kh năng sau: • N u n ch n, n = 2m. Gi s m = (bibi-1...b1)2, khi ñó n = 2m = (bibi-1...b10)2 ⇒ f(n) = f((bibi-1...b10)2) = f(2m) = f(m) = f((bibi-1...b1)2) = (b1b2 ...bi)2 = (0b1b2 ...bi)2 ⇒ (*) ñúng. • N u n l và n = 4m + 1. Gi s m = (bibi-1...b1)2, khi ñó n = (bibi-1...b101)2 ⇒ f(n) = f((bibi-1...b101)2) = f(4m+1) = 2.f(2m+1) - f(m) = 2.f((bibi-1...b11)2) - f((bibi-1...b1)2) = (10)2.(1b1b2 ...bi)2 - ( b1b2 ...bi)2 = (1b1b2 ...bi0)2 - ( b1b2 ...bi)2 = (10b1b2 ...bi)2 ⇒ (*) ñúng. 26
  • 27. • N u n l và n = 4m + 3. Gi s m = (bibi-1...b1)2, khi ñó n = (bibi-1...b111)2 ⇒ f(n) = f((bibi-1...b111)2) = f(4m+3) = 3f(2m+1) - 2f(m) = 3f((bibi-1...b11)2) - 2f((bibi-1...b1)2) = (11)2.(1b1b2 ...bi)2 - (10)2.(b1b2 ...bi)2 = (11b1b2 ...bi)2 ⇒ (*) ñúng. V y (*) ñúng và hàm f ñư c xác ñ nh như (*). Ví d 2 (Trích ñ thi c a Trung Qu c): Tìm hàm s f: N* → N* th a mãn: 1) f(1) =1; 2) f(2n) < 6f(n); 3) 3f(n)f(2n+1) = f(2n)(3f(n)+1) ∀n∈N*. L i gi i: Vì f(n)∈N* nên (3f(n), 3f(n)+1) = 1. T 3) suy ra 3f(n) | f(2n). K t h p v i 2) suy ra * f(2n) = 3f(n) và f(2n+1) = 3f(n)+1 ∀n∈N . Th m t s giá tr ta th y f(n) ñư c xác ñ nh như sau: “V i n = (b1b2…bi)2 thì f(n) = (b1b2…bi)3 ∀n∈N*” (*). Ta ch ng minh (*) b ng quy n p. + V i n = 1, 2, 3, 4 thì hi n nhiên (*) ñúng. + Gi s (*) ñúng cho k < n (v i n ≥ 4). Ta ch ng minh (*) ñúng cho n. • N u n ch n: n = 2m. Gi s m = (c1c2…cj)2 thì n = 2m = (c1c2…cj0)2. Khi ñó: f(n) = f(2m) = 3f(m) = 3.f((c1c2…cj)2) = (10)3.(c1c2…cj)3 = (c1c2…cj0)3 ⇒ (*) ñúng cho n ch n. • N u n l : n = 2m + 1 ⇒ n = (c1c2…cj1)2. Khi ñó: f(n) = f(2m+1) = 3f(m) + 1 = 3f((c1c2…cj)2) + 1 = (10)3.(c1c2…cj)3 + 13 = (c1c2…cj1)3 ⇒ (*) ñúng cho n l . V y (*) ñúng cho m i n∈N* và f(n) ñư c xác ñ nh như (*). Phương pháp 9: phương pháp s d ng ñ o hàm. Ví d 1: Tìm f: R → R tho mãn: | f(x)- f(y)|2 ≤ | x- y|3 ∀x, y∈R (14). L i gi i: C ñ nh y, v i x∈R, x ≠ y t (14) ta ñư c: 2 f ( x) − f ( y ) f ( x) − f ( y ) ≤ x− y ⇒0≤ ≤ x − y . Vì lim 0 = lim x − y = 0 nên suy ra x− y x− y x→ y x→ y f ( x) − f ( y ) lim = 0 ⇒ f’(y) = 0 ⇒ f(y) = c ∀y∈R (v i c là h ng s ). Th l i th y ñúng. x→ y x− y Ví d 2: Tìm f: R → R có ñ o hàm trên R và tho mãn: f(x+y) = f(x) + f(y) + 2xy ∀x, y∈R (15) L i gi i: + Cho x = y = 0 ta ñư c f(0) = 0. 27
  • 28. f ( x + y ) − f ( x) f ( y ) + 2 xy f ( y ) − f (0) + V i y ≠ 0, c ñ nh x ta ñư c: = = + 2 x (*). Vì f(x) y y y −0 có ñ o hàm trên R nên t (*), cho y → 0, suy ra f’(x) = f’(0) + 2x = 2x + c ⇒ f(x) = x2+cx+b ∀x∈R; b, c là các h ng s th c. Th l i th y ñúng. Phương pháp 10: phương pháp ñ t hàm ph . M c ñích chính c a vi c ñ t hàm ph là làm gi m ñ ph c t p c a phương trình hàm ban ñ u và chuy n ñ i tính ch t hàm s nh m có l i hơn trong gi i toán. Ví d 1: Tìm f: R → R tho mãn: f(x) ≥ 2007x và f(x+y) ≥ f(x)+f(y) ∀x, y∈R (1). L i gi i: D th y f(x) = 2007x là m t hàm s tho mãn (1). ð t g(x) = f(x) - 2007x và thay vào (1) ta ñư c: g(x) ≥ 0 (a) và g(x+y) ≥ g(x) + g(y) (b) ∀x, y∈R. + Cho x = y = 0, t (b) ta ñư c g(0) ≤ 0, k t h p v i (a) suy ra g(0) = 0. + Cho x = -y, x∈R, t (a) và (b) ta ñư c g(x) ≥ 0, g(-x) ≥ 0, 0 ≥ g(x) + g(-x); suy ra : g(x) = g(-x) = 0 ⇒ h(x) = 2007x, ∀x∈R. Th l i th y ñúng. Ví d 2: Tìm f: R → R liên t c trên R tho mãn: f(x+y) = f(x) + f(y) + f(x)f(y) ∀x, y∈ R (2). L i gi i: Xét phương trình: λ = 2λ + λ2 có nghi m λ = -1 khác 0. ð t g(x) = f(x) - (-1) = f(x) + 1. Th vào (18) ta ñư c: g(x+y) = g(x).g(y) ∀x, y∈R (*). t Cho x = y = ta ñư c g(t) ≥ 0 ∀t∈R. 2 Cho x = y = 0 ta ñư c: g(0) = 0 ho c g(0) = 1. + N u g(0) = 0 thì (*) suy ra g(x) = 0 ∀x∈R ⇒ f(x) = -1 ∀x∈R. Th l i th y ñúng. + N u g(0) = 1: Gi s t n t i a ñ g(a) = 0 thì (*) suy ra g(x) = 0 ∀x∈R. Trái v i gi thi t g(0) = 1. V y g(x) > 0 ∀x∈R. ð t h(x) = lng(x) ta ñư c : h(x+y) = h(x) + h(y) (**). T f(x) liên t c trên R suy ra h(x) liên t c trên R. Theo phương trình hàm Côsi ta ñư c h(x) = cx (v i c là h ng s ) ⇒ f(x) = ecx - 1 ∀x∈R. Khi c = 0 thì f(x) = -1. V y trong m i trư ng h p f(x) = ecx - 1 ∀x∈R th l i th y ñúng. Phương pháp 11: S d ng tính liên t c c a hàm s . S d ng tính liên t c c a hàm s có 3 con ñư ng chính: Xây d ng bi n t N ñ n R, ch ng minh hàm s là h ng s , s d ng phương trình hàm Côsi. Ví d 1 (xây d ng bi n t N ñ n R): Tìm hàm f : R → R th a mãn: 1) f(x) liên t c trên R; 2) f(1) = 2; 28
  • 29. 3) f(xy) = f(x)f(y) - f(x+y) +1 ∀x,y∈R. L i gi i: Cho x = y = 0 ta ñư c: f(0) = 1. Cho x = 1, y∈R ta ñư c: f(y+1) = f(y) +1 (a). T f(0) = 1, f(1) = 2 và (a) quy n p ta suy ra f(n) = n+1 ∀n∈N. V i n∈N, (a) ⇒ f(-n) = f(-n+1) - 1 = f(-n+2) - 2 =…= f(0) -n = -n + 1. V y f(z) = z +1 ∀z∈Z. 1 1 1 V i ∀n∈N*, 2 = f(1) = f (n. ) = f (n) f ( ) − f (n + ) + 1 (b). M t khác t (a) ta có: n n n 1 1 1 1 f (n + ) = 1 + f (n − 1 + ) = 2 + f (n − 2 + ) = ... = n + f ( ) . Th vào (b) ta ñư c: n n n n 1 1 f ( ) = + 1. n n m m 1 1 1 V i q ∈ ℚ, q = , m ∈ ℤ, n ∈ ℕ* ta có: f (q ) = f ( ) = f (m. ) = f (m) f ( ) − f (m + ) + 1 = n n n n n 1 1 = (m + 1)( + 1) − f (m + ) + 1 (c). T (a) ta d dàng ch ng minh ñư c: n n 1 1 f (m + ) = m + f ( ) . Th vào (c) ta ñư c f(q) = q +1 ∀q∈Q. n n V i r∈R, t n t i dãy {rn} v i rn∈Q th a mãn lim rn = r . Khi ñó, do f liên t c nên ta có: f(r) = f(limrn) = limf(rn) = lim(rn+1) = limrn + 1 = r + 1. V y f(x) = x + 1 ∀x∈R. Th l i th y ñúng. Ví d 2 (ch ng minh hàm s là h ng s ): 1 1 Tìm hàm f: [0; ] → [0; ] th a mãn: 2 2 1 1) f(x) liên t c trên [0; ] 2 1  1 2) f ( x ) = f ( x 2 + ) ∀x ∈  0;  . 4  2 L i gi i:  x0 = a 1  V i a∈[0; ], xét dãy s :  2 1 . 2  xn+1 = xn + ∀n ∈ ℕ  4 D ch ng minh {xn} không âm (a). 1 1 1 1 x0 ≤ ⇒ x1 ≤ x0 + ≤ . Quy n p suy ra xn ≤ (b). 2 2 4 2 2 29
  • 30. 1 xn +1 − xn = ( xn − ) 2 ≥ 0 ⇒ xn +1 ≥ xn ∀n ∈ ℕ (c). 2 1 1 T (a), (b), (c) suy ra xn∈[0; ] và {xn} có gi i h n h u h n là limx n = . 2 2 1 1 V y v i m i a∈[0; ], f(a) = f(x1) = f(x2) =…= limf(xn) = f(limxn) = f( ) = c (c là h ng s ). 2 2 Th l i th y ñúng. Ví d 3 (s d ng phương trình hàm côsi - VMO năm 2006(b ng B)): Tìm f: R → R liên t c trên R tho mãn: f(x-y).f(y-z).f(z-x)+8 = 0 ∀x, y, z∈R (3). L i gi i: −8 Cho x = t, z = -t, y = 0, x∈ R ta ñư c: f(t).f(t).f(-2t) = -8 ⇒ f (−2t ) = < 0 ⇒ f(t) < 0 ( f (t )) 2 ∀t∈R. ð t g(x) = ln( f ( x) ) ⇒ f ( x) = −2.e g ( x) . Th vào (3) ta ñư c: −2 g(x-y)+g(y-z)+g(z-x) -8.e = -8 ⇔ g(x-y) + g(y-z) + g(z-x) = 0 (*). + Cho x = y = z = 0, t (*) ta ñư c g(0) = 0 (a). + Cho y = z = 0, x∈R, t (a) ta ñư c g(x) = g(-x) (b). T (*) và (b) suy ra g(x-y) + g(y-z) = -g(z-x) = -g(x-z) = g(x-y+y-z) ⇒ g(t+t’) = g(t) + g(t’) ∀t, t’∈R (**). Vì f liên t c trên R nên g(x) cũng liên t c trên R. T (**), theo phương trình hàm Côsi ta ñư c g(x) = ax ⇒ f(x) = -2.eax = -2.bx (V i b = ea > 0). Th l i th y ñúng. ------------------------------------------------------H t------------------------------------------------------- 30